Complex Final Exam (Cardiac, Respiratory, Neuro, GI, Renal, Disaster)

Ace your homework & exams now with Quizwiz!

A pt is seen in the HCP office and dx with community-acquired pneumonia. The nurse knows the most common symptoms that this pt may have is A. dyspnea B. abdominal pain C. back pain D. hypoxemia E. chest discomfort F. a smoker

A, D, E dyspnea hypoxemia chest discomfort

Which of the following treatment goals is best for the client with status asthmaticus? A. Avoiding intubation B. Determining the cause of the attack C. Improving exercise tolerance D. Reducing secretions

A. Avoiding intubation

Exercise has which of the following effects on clients with asthma, chronic bronchitis, and emphysema? A. It enhances cardiovascular fitness. B. It improves respiratory muscle strength. C. It reduces the number of acute attacks. D. It worsens respiratory function and is discouraged.

A. It enhances cardiovascular fitness Exercise can improve cardiovascular fitness & help the client tolerate periods of hypoxia better, perhaps reducing the risk of heart attack.

A patient has been admitted to the hospital with GI bleeding. Which is a priority nursing action for this patient? A. Obtain complete vital signs. B. Administer prescribed medication for pain. C. Administer prescribed antacids every 2 hours. D. Administer prescribed medication for nausea and vomiting

A. Obtain complete vital signs.

Clients with chronic obstructive bronchitis are given diuretic therapy. Which of the following reasons best explains why? A. Reducing fluid volume reduces oxygen demand B. Reducing fluid volume improves client's mobility C. Restricting fluid volume reduces sputum production D. Reducing fluid volume improves respiratory function

A. Reducing fluid volume reduces oxygen demand Reducing fluid volume reduces the workload of the heart, which reduces oxygen demand & in turn, reduces the RR.

The nurse working in the emergency department assesses a client who has symptoms of stroke. For what modifiable risk factors should the nurse assess? (Select all that apply.) a. Alcohol intake b. Diabetes c. High-fat diet d. Obesity e. Smoking

ANS: A, C, D, E Alcohol intake, a high-fat diet, obesity, and smoking are all modifiable risk factors for stroke. Diabetes is not modifiable but is a risk factor that can be controlled with medical intervention.

The nurse identifies the priority nursing dx for a pneumonia pt to be ineffective airway clearance related to fatigue, CP, excessive secretions and muscle weakness. To correct the problem the nurse will implement which intervention A. administer oxygen to prevent hypoxemia and atelectasis B. push fluids to greater than 3000 mL/day to ensure adequate hydration C. administer bronchodilator therapy in a timely manner to decrease bronchospasms D. maintain semi-fowlers position to facilitate breathing and prevent further fatigue

C bronchodilator

A patient has a systemic blood pressure (BP) of 120/60 mm Hg and an intracranial pressure of 24 mm Hg. The nurse determines that the cerebral perfusion pressure (CPP) of this patient indicates a. high blood flow to the brain. b. normal intracranial pressure (ICP). c. impaired brain blood flow. d. adequate cerebral perfusion.

Correct Answer: C Rationale: The patient's CPP is 56, below the normal of 70 to 100 mm Hg and approaching the level of ischemia and neuronal death. The patient has low cerebral blood flow/perfusion. Normal ICP is 0 to 15 mm Hg. Cognitive Level: Application Text Reference: p. 1468 Nursing Process: Assessment NCLEX: Physiological Integrity

The nurse notices a visitor walking into the room of a pt on airborne isolation with no protective gear. What does the nurse do? A. Ensures that the pt is wearing a mask B. Tells the visitor that the pt cannot receive visitors at this time C. Gives a particulate air respirator to the visitor D. Gives a mask to the visitor

D Because the visitor is entering the pt's isolation environment, the visitor must wear a mask. The pt typically must wear a mask only when he or she is outside of an isolation environment. Turning the visitor away is inappropriate and unnecessary. It would not be necessary for the visitor to wear an air respirator.

The nurse is reviewing a pneumonia pts lab results. What does she expect to see A. decreased Hgb B. increased RBCs C. decreased neutrophils D. increased WBCs

D incr WBCs

Your patient presents to the E.D. with severe pain in the upper chest while lying down, shortness of breath, and hoarseness while talking. The neck veins appear distended. He states that he has never had pain like this before. What problem do you expect your patient to have? A) MI B) Atelectasis C) Blindness D) Thoracic Aneurysm

D. These are all classic symptoms of a thoracic aneurysm

A patient's vomitus is dark brown and has a coffee-ground appearance. the nurse recognizes that this emsis is charactristic of: a. stomach bleeding b. an intestinal obstruction c. bile reflux d. active bleeding of lower esophagus

a. stomach bleeding Gastric bleed looks like coffee grounds, when blood combines with acid, it clots and causes partial decomposition

The nurse would recognize cardiogenic shock from other types of shock by the data collection findings for which of the following? a. Bronchospasm b. Oliguria c. Pulmonary edema d. Tachypnea

c. Pulmonary edema The presence of pulmonary edema is what differentiates cardiogenic shock from other forms of shock.

The client is admitted to the emergency department with chest trauma. When assess- ing the client, which signs/symptoms would the nurse expect to find that support the diagnosis of pneumothorax? 1. Bronchovesicular lung sounds and bradypnea. 2. Unequal lung expansion and dyspnea. 3. Frothy bloody sputum and consolidation. 4. Barrel chest and polycythemia.

***2. Unequal lung expansion and dyspnea would indicate a pneumothorax. TEST-TAKING HINT: The test taker can use "chest trauma" or "pneumothorax" to help select the correct answer. Both of these words should cause the test taker to select "2" because unequal chest expansion would result from trauma.

A patient with cardiomyopathy is demonstrating signs of cardiogenic shock. The nurse realizes that this type of shock is due to: 1. Reduced cardiac output 2. Increased stroke volume 3. Reduced blood volume 4. Blood flow blocked in the pulmonary circulation

1. Reduced cardiac output In cardiogenic shock, cardiac output is reduced, leading to poor tissue perfusion.

The client with an intestinal obstruction continues to have acute pain even though the nasoenteric tube is patent and draining. Which action by the nurse would be most appropriate? 1. Reassure the client that the nasoenteric tube is functioning. 2. Assess the client for a rigid abdomen. 3. Administer an opioid as ordered. 4. Reposition the client on the left side.

2. The client's pain may be indicative of peritonitis, and the nurse should assess for signs and symptoms, such as a rigid abdomen, elevated temperature, and increasing pain. Reassuring the client is important, but accurate assessment of the client is essential. The full assessment should occur before pain relief measures are employed. Repositioning the client to the left side will not resolve the pain.

A client is diagnosed with cardiogenic shock. The nurse should plan interventions to address which of the following potential complications of this disorder? 1. Pulmonary embolism 2. Deep vein thrombosis 3. Renal failure 4. Myocardial infarction

4. Myocardial infarction In cardiogenic shock, there is a reduction in oxygenated arterial blood that decreases perfusion throughout the body. The most serious complication of cardiogenic shock is myocardial infarction. Pulmonary embolism, deep vein thrombosis, and renal failure are not considered the most serious complications of cardiogenic shock.

A pt is being admitted for pneumonia. The sputum culture is positive for streptococcus, and the pt asks the nurse how long the treatment will be. The nurse knows? A. The pt will be treated for 5 to 7 days. B. The pt will require IV antibiotics for 7 to 10 days. C. The pt will complete 6 days of therapy. D. The pt must be afebrile for 24 hours.

A Anti-infectives usually are used for 5 to 7 days in uncomplicated community-acquired pneumonia, and for up to 21 days in an immunocompromised pt or one with hospital-acquired pneumonia

Emergency medical technicians arrive at the emergency department with an unresponsive client who has an oxygen mask in place. Which action should the nurse take first? a. Assess that the client is breathing adequately. b. Insert a large-bore intravenous line. c. Place the client on a cardiac monitor. d. Assess for the best neurologic response.

A ~ The highest-priority intervention in the primary survey is to establish that the client is breathing adequately. Even though this client has an oxygen mask on, he or she may not be breathing, or may be breathing inadequately with the device in place.

A patient has ARDS resulting from sepsis. Which measure is most likely to be implemented to maintain cardiac output? A. Administer crystalloid fluids or colloid solutions. B. Position the patient in the Trendelenburg position. C. Perform chest physiotherapy and assist with staged coughing. D. Place the patient on fluid restriction, and administer diuretics.

A. Administer crystalloid fluids or colloid solutions. Low cardiac output may necessitate crystalloid fluids or colloid solutions in addition to lowering the PEEP or administering inotropes. The Trendelenburg position (not recommended to treat hypotension) and chest physiotherapy are unlikely to relieve decreased cardiac output, and fluid restriction and diuresis are inappropriate interventions.

What are the most common early clinical manifestations of ARDS? A. Dyspnea and tachypnea B. Cyanosis and apprehension C. Hypotension and tachycardia D. Respiratory distress and frothy sputum

A. Dyspnea and tachypnea The initial manifestations of ARDS are often subtle. At the time of the initial injury and for several hours up to 2 days afterward, the patient may not experience respiratory symptoms, or the patient may exhibit only dyspnea, tachypnea, cough, and restlessness.

After a stroke, a client has ataxia. What intervention is most appropriate to include on the client's plan of care? a. Ambulate only with a gait belt. b. Encourage double swallowing. c. Monitor lung sounds after eating. d. Perform post-void residuals.

ANS: A Ataxia is a gait disturbance. For the client's safety, he or she should have assistance and use a gait belt when ambulating. Ataxia is not related to swallowing, aspiration, or voiding.

A client appears dyspneic, but the oxygen saturation is 97%. What action by the nurse is best? a. Assess for other manifestations of hypoxia. b. Change the sensor on the pulse oximeter. c. Obtain a new oximeter from central supply. d. Tell the client to take slow, deep breaths.

ANS: A Pulse oximetry is not always the most accurate assessment tool for hypoxia as many factors can interfere, producing normal or near-normal readings in the setting of hypoxia. The nurse should conduct a more thorough assessment. The other actions are not appropriate for a hypoxic client.

A nursing student studying acute coronary syndromes learns that the pain of a myocardial infarction (MI) differs from stable angina in what ways? (Select all that apply.) a. Accompanied by shortness of breath b. Feelings of fear or anxiety c. Lasts less than 15 minutes d. No relief from taking nitroglycerin e. Pain occurs without known cause

ANS: A, B, D, E The pain from an MI is often accompanied by shortness of breath and fear or anxiety. It lasts longer than 15 minutes and is not relieved by nitroglycerin. It occurs without a known cause such as exertion.

A client is having a peritoneal dialysis treatment. The nurse notes an opaque color to the effluent. What is the priority action by the nurse? a. Warm the dialysate solution in a microwave before instillation. b. Take a sample of the effluent and send to the laboratory. c. Flush the tubing with normal saline to maintain patency of the catheter. d. Check the peritoneal catheter for kinking and curling.

ANS: B An opaque or cloudy effluent is the first sign of peritonitis. A sample of the effluent would need to be sent to the laboratory for culture and sensitivity in order to administer the correct antibiotic. Warming the dialysate in a microwave and flushing the tubing are not safe actions by the nurse. Checking the catheter for obstruction is a viable option but will not treat the peritonitis.

After teaching a client with nephrotic syndrome and a normal glomerular filtration, the nurse assesses the client's understanding. Which statement made by the client indicates a correct understanding of the nutritional therapy for this condition? a. "I must decrease my intake of fat." b. "I will increase my intake of protein." c. "A decreased intake of carbohydrates will be required." d. "An increased intake of vitamin C is necessary."

ANS: B In nephrotic syndrome, the renal loss of protein is significant, leading to hypoalbuminemia and edema formation. If glomerular filtration is normal or near normal, increased protein loss should be matched by increased intake of protein. The client would not need to adjust fat, carbohydrates, or vitamins based on this disorder.

A nurse assesses a client admitted to the cardiac unit. Which statement by the client alerts the nurse to the possibility of right-sided heart failure? a. "I sleep with four pillows at night." b. "My shoes fit really tight lately." c. "I wake up coughing every night." d. "I have trouble catching my breath."

ANS: B Signs of systemic congestion occur with right-sided heart failure. Fluid is retained, pressure builds in the venous system, and peripheral edema develops. Left-sided heart failure symptoms include respiratory symptoms. Orthopnea, coughing, and difficulty breathing all could be results of left-sided heart failure.

An emergency room nurse assesses a client with potential liver trauma. Which clinical manifestations should alert the nurse to internal bleeding and hypovolemic shock? (Select all that apply.) a. Hypertension b. Tachycardia c. Flushed skin d. Confusion e. Shallow respirations

ANS: B, D Symptoms of hemorrhage and hypovolemic shock include hypotension, tachycardia, tachypnea, pallor, diaphoresis, cool and clammy skin, and confusion.

A client has an intra-arterial blood pressure monitoring line. The nurse notes bright red blood on the client's sheets. What action should the nurse perform first? a. Assess the insertion site. b. Change the client's sheets. c. Put on a pair of gloves. d. Assess blood pressure.

ANS: C For the nurse's safety, he or she should put on a pair of gloves to prevent blood exposure. The other actions are appropriate as well, but first the nurse must don a pair of gloves.

A client has been brought to the emergency department with a life-threatening chest injury. What action by the nurse takes priority? a. Apply oxygen at 100%. b. Assess the respiratory rate. c. Ensure a patent airway. d. Start two large-bore IV lines.

ANS: C The priority for any chest trauma client is airway, breathing, circulation. The nurse first ensures the client has a patent airway. Assessing respiratory rate and applying oxygen are next, followed by inserting IVs.

A nurse is caring for a client who is scheduled for a dose of cefazolin and vitamins at this time. Hemodialysis for this client is also scheduled in 60 minutes. Which action by the nurse is best? a. Administer cefazolin since the level of the antibiotic must be maintained. b. Hold the vitamins but administer the cefazolin. c. Hold the cefazolin but administer the vitamins. d. Hold all medications since both cefazolin and vitamins are dialyzable.

ANS: D Both the cefazolin and the vitamins should be held until after the hemodialysis is completed because they would otherwise be removed by the dialysis process.

A nurse is caring for a client on mechanical ventilation. When double-checking the ventilator settings with the respiratory therapist, what should the nurse ensure as a priority? a. The client is able to initiate spontaneous breaths. b. The inspired oxygen has adequate humidification. c. The upper peak airway pressure limit alarm is off. d. The upper peak airway pressure limit alarm is on.

ANS: D The upper peak airway pressure limit alarm will sound when the airway pressure reaches a preset maximum. This is critical to prevent damage to the lungs. Alarms should never be turned off. Initiating spontaneous breathing is important for some modes of ventilation but not others. Adequate humidification is important but does not take priority over preventing injury.

To inflate the cuff of an endotracheal tube (ET) when the patient is on mechanical ventilation, the nurse a. inflates the cuff until the pilot balloon is firm. b. inflates the cuff with a minimum of 10 mL of air. c. injects air into the cuff until a manometer shows 15 mm Hg pressure. d. injects air into the cuff until a slight leak is heard only at peak inflation.

ANS: D The minimal occluding volume technique involves injecting air into the cuff until an air leak is present only at peak inflation. The volume to inflate the cuff varies with the ET and the patient's size. Cuff pressure should be maintained at 20 to 25 mm Hg. An accurate assessment of cuff pressure cannot be obtained by palpating the pilot balloon. DIF: Cognitive Level: Comprehension REF: 1701-1702 TOP: Nursing Process: Implementation MSC: NCLEX: Physiological Integrity

Surgical management of ulcerative colitis may be performed to treat which of the following complications? 1. Gastritis 2. Bowel herniation 3. Bowel outpouching 4. Bowel perforation

Answer: 4. Bowel perforation Perforation, obstruction, hemorrhage, and toxic megacolon are common complications of ulcerative colitis that may require surgery. Herniation and gastritis aren't associated with irritable bowel diseases, and outpouching of the bowel is diverticulosis.

A nurse evaluates the blood theophylline level of a client receiving aminophylline (theophylline) by intravenous infusion. The nurse would determine that a therapeutic blood level exists if which of the following were noted in the laboratory report? A. 5 mcg/mL B. 15 mcg/mL C. 25 mcg/mL D. 30 mcg/mL

B The therapeutic theophylline blood level range from 10-20 mcg/mL.

The clinical instructor is reviewing common complications of pneumonia with the students. She knows that further instruction is needed when the students identify which of these? A. sepsis B. ventilation/perfusion issues C. hypoxemia D. pleural effusion E. respiratory failure F. atelectasis

B pneumonia is a ventilation problem not perfusion

Which of the following are signs of a rupturing AAA? Select all that apply: A) Increased BP B) Decreased Hct C) Low Back Pain D) Decreased BP E) Intermittent abdominal pain

B,C, and D. The patient is losing blood pressure and dropping hematocrit due to bleeding. Pain is a key sign of a rupturing (or close to rupturing) Aortic Abdominal Aneurysm. Usually, when rupturing, the pain becomes constant rather than intermittent.

A client with acute asthma showing inspiratory & expiratory wheezes & a decreased expiratory volume should be treated with which of the following classes of medication right away? A. Beta- adrenergic blockers B. Bronchodilators C. Inhaled steroids D. Oral steroids

B. Bronchodilators Bronchodilators are the first line of treatment for asthma because bronchocontriction is the cause of reduced airflow.

A 19-year-old comes into the emergency department with acute asthma. His respiratory rate is 44 breaths/minute, and he appears to be in acute respiratory distress. Which of the following actions should be taken first? A. Take a full medication history B. Give a bronchodilator by nebulizer C. Apply a cardiac monitor to the client D. Provide emotional support to the client

B. Give a bronchodilator by nebulizer The client is having an acute asthma attack & needs to increase oxygen delivery to the lung & body. Nebulized bronchodilators open airways & increase the amount of oxygen delivered.

Dani was given dilaudid for pain. She's sleeping & her RR is 4 bpm. If action isn't taken quickly, she might have which of the following reactions? A. Asthma attack B. Respiratory arrest C. Be pissed about receiving Narcan D. Wake up on her own

B. Respiratory attack Narcotics can cause respiratory arrest if given in large doses.

Which of the following aneurysms is the most likely to dissect? A) Abdominal Aortic B) Thoracic C) Peripheral D) They are all equally likely

B. The thoracic region of the aorta often has the highest pressure and force due to being closest to the heart.

The nurse suspects the presence of an arterial epidural hematoma in the patient who experiences a. failure to regain consciousness following a head injury b. a rapid deterioration of neurologic function within 24 to 48 hours following a head injury c. nonspecific, nonlocalizing progression of alteration in LOC occurring over weeks or months d. unconsciousness at the time of a head injury with a brief period of consciousness followed by a decrease in LOC

D. An arterial epidural hematoma is the most acute neurologic emergency, and the typical symptoms include unconsciousness at the scene, with a brief lucid interval followed by a decrease in LOC. An acute subdural hematoma manifests signs within 48 hours of an injury; a chronic subdural hematoma develops over weeks or months

The oxygen delivery system chosen for the patient in acute respiratory failure should A. always be a low-flow device, such as a nasal cannula. B. correct the PaO2 to a normal level as quickly as possible. C. administer positive-pressure ventilation to prevent CO2 narcosis. D. maintain the PaO2 at ≥60 mm Hg at the lowest O2 concentration possible.

D. maintain the PaO2 at ≥60 mm Hg at the lowest O2 concentration possible. The selected oxygen delivery system must maintain PaO2 at 55 to 60 mm Hg and SaO2 at 90% or greater at the lowest oxygen concentration possible.

A critical concern for a post op pt returning to the floor is r/t impaired oxygenation caused by inadequate ventilation. The ABG and assessment finding that alerts the nurse to use oxygen and the ICS is A. PaO2 is 90mm Hg with crackles B. PaO2 is 45mm Hg with atelectasis C. PaO2 is 90mm Hg with wheezing D. PaO2 is 38mm Hg with clear lung sounds

A PaO2 is 90mm Hg with crackles

A nurse is auscultating the lower lung fields of a pneumonia pt. She hears coarse crackles and ids the problem as impaired oxygenation. She knows the underlying physiologic condition associated with pt condition is A. hypoxemia B. hyperemia C. hypocapnia D. hypercapnia

A hypoxemia

Which pt is at higher risk for developing pneumonia? A. any hospitalized pt between 19 - 64 y.o. B. 36 y.o. trauma pt on mechanical ventilator C. disabled 51 y.o. with abdominal pain, d/c home D. Any pt who has not received the pneumonia vaccine

B 36 y.o. trauma pt on mechanical ventilator

The nurse knows, in caring for her ventilated pt, that the goal is to prevent VAP. She identifies the following as important in reaching this goal. Select All That Apply A. she should avoid wearing jewelry B. HOB should be up C. Administer chest physiotherapy D. Provide oral care every 12 hours E. Hand hygeine

A, B, E chest physiotherapy is not an intervention for this pt oral care should be provided every 8 hours, not 12 hours

An emergency department (ED) nurse is preparing to transfer a client to the trauma intensive care unit. Which information should the nurse include in the nurse-to-nurse hand-off report? (SATA) a. Mechanism of injury b. Diagnostic test results c. Immunizations d. List of home medications e. Isolation precautions

A, B, E ~ Hand-off communication should be comprehensive so that the receiving nurse can continue care for the client fluidly. Communication should be concise and should include only the most essential information for a safe transition in care. Hand-off communication should include the clients situation (reason for being in the ED), brief medical history, assessment and diagnostic findings, Transmission-Based Precautions needed, interventions provided, and response to those interventions.

Which conditions does the nurse recognize as a risk for developing aspiration pneumonia? Select All That Apply A. continuous tube feed B. bronchoscopy procedure C. MRI D. decreased LOC E. stroke F. chest tube

A, B. D. E continuous tube feed bronchoscopy procedure decreased LOC stroke

You are providing care for an elderly patient who has a low PaO2 as a result of worsening left-sided pneumonia. Which nursing intervention will help the patient mobilize his secretions? A. Augmented coughing or huff coughing B. Positioning the patient to lie on his left side C. Frequent and aggressive nasopharyngeal suctioning D. Application of noninvasive positive-pressure ventilation (NIPPV)

A. Augmented coughing or huff coughing Augmented coughing and huff coughing techniques may aid the patient in the mobilization of secretions. If placed in a side-lying position, the patient should be positioned on his right side (good lung down). Suctioning may be indicated, but it should always be performed cautiously because of the risk of hypoxia. NIPPV is inappropriate in the treatment of patients with excessive secretions.

A nurse is caring for four clients in the neurologic intensive care unit. After receiving the hand-off report, which client should the nurse see first? a. Client with a Glasgow Coma Scale score that was 10 and is now is 8 b. Client with a Glasgow Coma Scale score that was 9 and is now is 12 c. Client with a moderate brain injury who is amnesic for the event d. Client who is requesting pain medication for a headache

ANS: A A 2-point decrease in the Glasgow Coma Scale score is clinically significant and the nurse needs to see this client first. An improvement in the score is a good sign. Amnesia is an expected finding with brain injuries, so this client is lower priority. The client requesting pain medication should be seen after the one with the declining Glasgow Coma Scale score.

A client is on a ventilator and is sedated. What care may the nurse delegate to the unlicensed assistive personnel (UAP)? a. Assess the client for sedation needs. b. Get family permission for restraints. c. Provide frequent oral care per protocol. d. Use nonverbal pain assessment tools.

ANS: C The client on mechanical ventilation needs frequent oral care, which can be delegated to the UAP. The other actions fall within the scope of practice of the nurse.

The emergency department team is performing cardiopulmonary resuscitation on a client when the client's spouse arrives at the emergency department. Which action should the nurse take first? a. Request that the client's spouse sit in the waiting room. b. Ask the spouse if he wishes to be present during the resuscitation. c. Suggest that the spouse begin to pray for the client. d. Refer the client's spouse to the hospitals crisis team.

B ~ If resuscitation efforts are still under way when the family arrives, one or two family members may be given the opportunity to be present during lifesaving procedures. The other options do not give the spouse the opportunity to be present for the client or to begin to have closure.

What distinguishes hypercapnic respiratory failure from hypoxemic respiratory failure? A. Low oxygen saturation despite administration of supplemental oxygen B. Acidemia for which the body cannot compensate C. Respiration rate greater than 30 breaths/minute D. Heart rate increases above 100 beats/minute

B. Acidemia for which the body cannot compensate Hypercapnic respiratory failure is PaCO2 greater than 48 mm Hg in combination with acidemia. The body cannot compensate for the acidemia. Hypoxemic respiratory failure is a PaO2 less than 60 mm Hg despite receiving an inspired oxygen concentration greater than or equal to 60%. The respiratory rate and heart rate are not part of the definitions of these two conditions.

Before administering sodium polystyrene sulfonate (Kayexalate) to a patient with hyperkalemia, the nurse should assess the a. blood urea nitrogen (BUN) and creatinine. b. blood glucose level. c. patient's bowel sounds. d. level of consciousness (LOC).

C Sodium polystyrene sulfonate (Kayexalate) should not be given to a patient with a paralytic ileus (as indicated by absent bowel sounds) because bowel necrosis can occur. The BUN and creatinine, blood glucose, and LOC would not affect the nurse's decision to give the medication.

An emergency department (ED) case manager is consulted for a client who is homeless. Which intervention should the case manager provide? a. Communicate client needs and restrictions to support staff. b. Prescribe low-cost antibiotics to treat community-acquired infection. c. Provide referrals to subsidized community-based health clinics. d. Offer counseling for substance abuse and mental health disorders.

C ~ Case management interventions include facilitating referrals to primary care providers who are accepting new clients or to subsidized community-based health clinics for clients or families in need of routine services. The ED nurse is accountable for communicating pertinent staff considerations, client needs, and restrictions to support staff (e.g., physical limitations, isolation precautions) to ensure that ongoing client and staff safety issues are addressed. The ED physician prescribes medications and treatments. The psychiatric nurse team evaluates clients with emotional behaviors or mental illness and facilitates the follow-up treatment plan, including possible admission to an appropriate psychiatric facility.

Which of the following types of asthma involves an acute asthma attack brought on by an upper respiratory infection? A. Emotional B. Extrinsic C. Intrinsic D. Mediated

C. Intrinsic Intrinsic asthma doesn't have an easily identifiable allergen & can be triggered by a common cold. Extrinsic asthma is caused by dust, molds, & pets.

The nurse notes that a patient with a head injury has a clear nasal drainage. The most appropriate nursing action for this finding is to a. obtain a specimen of the fluid and send for culture and sensitivity. b. take the patient's temperature to determine whether a fever is present. c. check the nasal drainage for glucose with a Dextrostik or Testape. d. have the patient to blow the nose and then check the nares for redness.

Correct Answer: C Rationale: If the drainage is cerebrospinal fluid (CSF) leakage from a dural tear, glucose will be present. Fluid leaking from the nose will have normal nasal flora, so culture and sensitivity will not be useful. A dural tear does increase the risk for infections such as meningitis, but the nurse should first determine whether the clear drainage is CSF. Blowing the nose is avoided to prevent CSF leakage. Cognitive Level: Application Text Reference: p. 1481 Nursing Process: Implementation NCLEX: Physiological Integrity

A client's ABG analysis reveals a pH of 7.18, PaCO2 of 72 mm Hg, PaO2 of 77 mm Hg, and HCO3- of 24 mEq/L. What do these values indicate? A. Metabolic acidosis B. Respiratory alkalosis C. Metabolic alkalosis D. Respiratory acidosis

D Respiratory Acidosis

The nurse is reviewing the lab results for an older adult pneumonia pt. The lab value frequently seen in pneumonia pts that may not be seen in this pt is A. RBC 4.0 - 5.0 B. Hgb 12 - 16 C. Hct 36 - 48 D. WBC 12 - 18

D WBC

Teaching for a client with chronic obstructive pulmonary disease (COPD) should include which of the following topics? A. How to have his wife learn to listen to his lungs with a stethoscope from Walmart B. How to increase his oxygen therapy C. How to treat respiratory infections without going to the physician D. How to recognize the signs of an impending respiratory infection

D. How to recognize the signs of an impending respiratory infection Respiratory infection in clients with a respiratory disorder can be fatal.

Which of the following measures can reduce or prevent the incidence of atelectasis in a post operative client? A. Chest physiotherapy B. Mechanical ventilation C. Reducing oxygen requirements D. Use of an incentive spirometer

D. Use of an incentive spirometer Using an incentive spirometer requires the client to take deep breaths & promotes lung expansion.

As the nurse you know that acute respiratory distress syndrome (ARDS) can be caused by direct or indirect lung injury. Select below all the INDIRECT causes of ARDS: A. Drowning B. Aspiration C. Sepsis D. Blood transfusion E. Pneumonia F. Pancreatitis

The answers are: C, D, F Indirect causes are processes that can cause inflammation OUTSIDE of the lungs....so the issue arises somewhere outside the lungs. Therefore, sepsis (infection...as long as it is outside the lungs), blood transfusion, and pancreatitis are INDIRECT causes. Drowning, aspiration, and pneumonia are issues that arise in the lungs (therefore, they are DIRECT causes of lung injury)

Dobutamine (Dobutrex) is used to treat a client experiencing cardiogenic shock. Nursing intervention includes: a.) Monitoring for fluid overload. b.) Monitoring for cardiac dysrhythmias. c.) Monitoring respiratory status. d.) Monitoring for hypotension.

b.) Monitoring for cardiac dysrhythmias. Rationale: Dobutamine is beneficial in cases where shock is caused by heart failure. The drug increases contractility, and has the potential to cause dysrhythmias.

Acute tubular necrosis (ATN) is the most common cause of intrarenal AKI. Which patient is most likely to develop ATN? a. Patient with diabetes mellitus b. Patient with hypertensive crisis c. Patient who tried to overdose on acetaminophen d. Patient with major surgery who required a blood transfusion

d. Acute tubular necrosis (ATN) is primarily the result of ischemia, nephrotoxins, or sepsis. Major surgery is most likely to cause severe kidney ischemia in the patient requiring a blood transfusion. A blood transfusion hemolytic reaction produces nephrotoxic injury if it occurs. Diabetes mellitus, hypertension, and acetaminophen overdose will not contribute to ATN.

A nurse is assessing a client who has been admitted with a diagnosis of an obstruction in the small intestine. The nurse should assess the client for? Select all that apply. 1. Projectile vomiting. 2. Significant abdominal distention. 3. Copious diarrhea. 4. Rapid onset of dehydration. 5. Increased bowel sounds

1, 4, 5. Signs and symptoms of intestinal obstructions in the small intestine may include projectile vomiting and rapidly developing dehydration and electrolyte imbalances. The client will also have increased bowel sounds, usually high-pitched and tinkling. The client would not normally have diarrhea and would have minimal abdominal distention. Pain is intermittent, being relieved by vomiting. Intestinal obstructions in the large intestine usually evolve slowly, produce persistent pain, and vomiting is less common. Clients with a large-intestine obstruction may develop obstipation and significant abdominal distention.

The nurse is preparing medications for a patient being treated for cardiogenic shock. Which medications will the nurse most likely provide to this patient? Note: Credit will be given only if all correct choices and no incorrect choices are selected. Select all that apply. 1. Dopamine 2. Norepinephrine 3. Dobutamine 4. Epinephrine 5. Phenylephrine

1. Dopamine 2. Norepinephrine 3. Dobutamine 5. Phenylephrine Rationale 1: Dopamine is commonly used in the treatment of cardiogenic shock. Rationale 2: Norepinephrine is commonly used in the treatment of cardiogenic shock. Rationale 3: Dobutamine may be used in the patient with cardiogenic shock who has an adequate blood pressure. Rationale 4: Epinephrine is not used in the treatment of cardiogenic shock. Rationale 5: Phenylephrine is a vasopressor and may be used in the patient with cardiogenic shock who is receiving dobutamine.

After receiving change-of-shift report in the coronary care unit, which client should you assess first? 1. The client with acute coronary syndrome who has a 3-pound weight gain and dyspnea 2. The client with percutaneous coronary angioplasty who has a dose of heparin scheduled 3. The client who had bradycardia after a myocardial infarction and now has a paced heart rate of 64 4. A client who has first-degree heart block, rate 68, after having an inferior myocardial infarction

1. Dyspnea and weight gain are symptoms of left ventricular failure and pulmonary edema; the client needs prompt intervention.

The nurse suspects that a patient diagnosed with a myocardial infarction is developing cardiogenic shock. What manifestation did the nurse assess to come to this conclusion? 1. jugular vein distention 2. warm extremities 3. laryngospasm 4. urticaria

1. jugular vein distention Jugular vein distention is seen in cardiogenic shock. Warm extremities are seen in early septic shock and anaphylactic shock. Laryngospasm and urticaria are seen in anaphylactic shock.

An older adult resident in a long-term-care facility becomes confused and agitated, telling the nurse, "Get out of here! You're going to kill me!" Which action will the nurse take first? A. Check the resident's oxygen saturation. B. Do a complete neurologic assessment. C. Give the prescribed PRN lorazepam (Ativan). D. Notify the resident's primary care provider

A A common reason for sudden confusion in older clients is hypoxemia caused by undiagnosed pneumonia. The nurse's first action should be to assess oxygenation by checking the pulse oximetry. Determining the cause of the confusion is the primary goal of the RN.

An elderly client with pneumonia may appear with which of the following symptoms first? A. Altered mental status and dehydration B. Fever and chills C. Hemoptysis and dyspnea D. Pleuritic chest pain and cough

A Fever, chills, hemoptysis, dyspnea, cough, and pleuritic chest pain are common symptoms of pneumonia, but elderly clients may first appear with only an altered mental status and dehydration due to a blunted immune response.

A nurse is caring for an 89-year-old client admitted with pneumonia. He has an IV of normal saline running at 100 mL/hr and antibiotics that were initiated in the emergency department 3 hours ago. He has oxygen at 2 liters/nasal cannula. What assessment finding by the nurse indicates that goals for a priority diagnosis have been met for this client? A. The client is alert and oriented to person, place, and time. B. Blood pressure is within normal limits and client's baseline. C. Skin behind the ears demonstrates no redness or irritation. D. Urine output has been >30 mL/hr per Foley catheter.

A Rationale: One of the first manifestations of pneumonia in an older adult is acute confusion as a result of impaired gas exchange. A client with pneumonia who is alert and oriented to person, place, and time is responding well to appropriate therapy for the disorder. The blood pressure is not an indicator of effective management of pneumonia, and neither is urine output. The skin behind his ears being intact is important and desirable but is not an outcome indicator for pneumonia management.

When auscultating the chest of a client with pneumonia, the nurse would expect to hear which of the following sounds over areas of consolidation? A. Bronchial B. Bronchovesicular C. Tubular D. Vesicular

A Bronchial Chest auscultation reveals bronchial breath sounds over areas of consolidation. Bronchovesicular are normal over midlobe lung regions, tubular sounds are commonly heard over large airways, and vesicular breath sounds are commonly heard in the bases of the lung fields.

An older adult resident in a long-term-care facility becomes confused and agitated, telling the nurse, "Get out of here! You're going to kill me!" Which action will the nurse take first? A) Check the resident's oxygen saturation. B) Do a complete neurologic assessment. C) Give the prescribed PRN lorazepam (Ativan). D) Notify the resident's primary care provider.

A) Check the resident's oxygen saturation.

The patient has developed cardiogenic shock and is decompensating. What pattern of hemodynamic alterations does the nurse expect to find? A) High preload, high afterload, low cardiac index, tachycardia B) Low preload, low afterload, high cardiac index, bradycardia C) Low preload, high afterload, high cardiac index, tachycardia D) High preload, low afterload, high cardiac index, tachycardia

A) High preload, high afterload, low cardiac index, tachycardia

A nurse is caring for an 89-year-old client admitted with pneumonia. He has an IV of normal saline running at 100 mL/hr and antibiotics that were initiated in the emergency department 3 hours ago. He has oxygen at 2 liters/nasal cannula. What assessment finding by the nurse indicates that goals for a priority diagnosis have been met for this client? A) The client is alert and oriented to person, place, and time. B) Blood pressure is within normal limits and client's baseline. C) Skin behind the ears demonstrates no redness or irritation. D) Urine output has been >30 mL/hr per Foley catheter

A) The client is alert and oriented to person, place, and time.

The nurse is caring for a pt who often coughs and chokes while eating and taking his meds, but the pt insists he is fine. The nurse recognizes this as a priority pt problem of risk for aspiration. The nursing interventions she would implement to prevent aspiration pneumonia are Select All That Apply A. HOB always elevated during meals B. Monitor pt ability to swallow small bites C. Give small frequent drinks of thin liquid D. Consult a nutritionist and obtain swallow studies E. Monitor the patient's ability to swallow saliva F. Place pt on NPO status til swallowing returns to normal

A, B, D, E

The nurse is assessing the stooling patterns of an assigned patient. The patient reports stools as being clay colored. The nurse knows this may indicate which condition? A.Bile is not reaching the intestines. B. The stool contains undigested fat. C. The stool has an excessive amount of bilirubin. D. The patient is experiencing upper gastrointestinal (GI) bleeding.

A.Bile is not reaching the intestines.

The pneumonia pt asks the nurse why she needs to draw labs to check his electrolytes. The nurses correct response is A. To monitor for possible acidosis B. To check for elevated WBCs C. To evaluate the sodium level for possible hypernatremia D. To check for possible septicemia

C Because of dehydration from fever, may be hypernatemic/ dehydrated Electrolytes don't show acidosis, that is ABGS It is important to check the WBC count but its not an electrolyte A blood culture would need to be done for septicemia

A patient is on mechanical ventilation with PEEP (positive end-expiratory pressure). Which finding below indicates the patient is developing a complication related to their therapy and requires immediate treatment? A. HCO3 26 mmHg B. Blood pressure 70/45 C. PaO2 80 mmHg D. PaCO2 38 mmHg

The answer is B. Mechanical ventilation with PEEP can cause issues with intrathoracic pressure and decrease the cardiac output (watch out for a low blood pressure) along with hyperinflation of the lungs (possible pneumothorax or subq emphysema which is air that escapes into the skin because the lungs are leaking air).

You're teaching a class on critical care concepts to a group of new nurses. You're discussing the topic of acute respiratory distress syndrome (ARDS). At the beginning of the lecture, you assess the new nurses understanding about this condition. Which statement by a new nurse demonstrates he understands the condition? A. "This condition develops because the exocrine glands start to work incorrectly leading to thick, copious mucous to collect in the alveoli sacs." B. "ARDS is a pulmonary disease that gradually causes chronic obstruction of airflow from the lungs." C. "Acute respiratory distress syndrome occurs due to the collapsing of a lung because air has accumulated in the pleural space." D. "This condition develops because alveolar capillary membrane permeability has changed leading to fluid collecting in the alveoli sacs."

The answer is D. ARDS is a type of respiratory failure that occurs when the capillary membrane that surrounds the alveoli sac becomes damaged, which causes fluid to leak into the alveoli sac. Option A describes cystic fibrosis, option B describes COPD, and option C describes a pneumothorax.

You're precepting a nursing student who is assisting you care for a patient on mechanical ventilation with PEEP for treatment of ARDS. The student asks you why the PEEP setting is at 10 mmHg. Your response is: A. "This pressure setting assists the patient with breathing in and out and helps improve air flow." B. "This pressure setting will help prevent a decrease in cardiac output and hyperinflation of the lungs." C. "This pressure setting helps prevent fluid from filling the alveoli sacs." D. "This pressure setting helps open the alveoli sacs that are collapsed during exhalation."

The answer is D. This setting of PEEP (it can range between 10 to 20 mmHg of water) and it helps to open the alveoli sacs that are collapsed, especially during exhalation.

When assessing the hemodynamic information for a newly admitted patient in shock of unknown etiology, the nurse will anticipate administration of large volumes of crystalloids when the a.) cardiac output is increased and the central venous pressure (CVP) is low. b.) pulmonary artery wedge pressure (PAWP) is increased, and the urine output is low. c.) heart rate is decreased, and the systemic vascular resistance is low. d.) cardiac output is decreased and the PAWP is high.

a.) cardiac output is increased and the central venous pressure (CVP) is low. Rationale: A high cardiac output and low CVP suggest septic shock, and massive fluid replacement is indicated. Increased PAWP indicates that the patient has excessive fluid volume (and suggests cardiogenic shock), and diuresis is indicated. Bradycardia and a low systemic vascular resistance (SVR) suggest neurogenic shock, and fluids should be infused cautiously.

Which intervention should the nurse implement first for the client diagnosed with a hemothorax who has had a right-sided chest tube for three (3) days and has no fluctu- ation (tidaling) in the water compartment? 1. Assess the client's bilateral lung sounds. 2. Obtain an order for a STAT chest x-ray. 3. Notify the health-care provider as soon as possible. 4. Document the findings in the client's chart.

***1. Assessment of the lung sounds could indi- cate that the client's lung has reexpanded because it has been three (3) days since the chest tube has been inserted. TEST-TAKING HINT: When the stem asks the test taker to identify the first intervention, all four (4) answer options could be interventions that are appropriate for the situation, but only one (1) is the first intervention. Remember to apply the nursing process: the first step is assessment.

Which action should the nurse implement for the client with a hemothorax who has a right-sided chest tube and there is excessive bubbling in the water-seal compartment? 1. Check the amount of wall suction being applied. 2. Assess the tubing for any blood clots. 3. Milk the tubing proximal to distal. 4. Encourage the client to cough forcefully.

***1. Checking to see if someone has increased the suction rate is the simplest action for the nurse to implement; if it is not on high, then the nurse must check to see if the problem is with the client or the system. TEST-TAKING HINT: The test taker should always think about assessing the client if there is a problem and the client is not in immediate danger. This would cause the test taker to eliminate options "3" and "4." If the test taker thinks about bubbling, he or she should know it has to do with suctioning.

Which intervention should the nurse implement for a male client who has had a left- sided chest tube for six (6) hours and refuses to take deep breaths because it hurts too much? 1. Medicate the client and have the client take deep breaths. 2. Encourage the client to take shallow breaths to help with the pain. 3. Explain that deep breaths do not have to be taken at this time. 4. Tell the client that if he doesn't take deep breaths, he could die.

***1. The client must take deep breaths to help push the air out of the pleural space into the water-seal drainage, and deep breaths will help prevent the client from develop- ing pneumonia or atelectasis. TEST-TAKING HINT: If the test taker reads options "2" and "3" and notices that both reflect the same idea namely, that deep breaths are not necessary then both can either be eliminated as incorrect answers or kept as possible correct answers. Option "4" should be eliminated based on being a very rude and threatening comment.

Which assessment data indicate that the chest tubes have been effective in treating the client with a hemothorax who has a right-sided chest tube? 1. There is gentle bubbling in the suction compartment. 2. There is no fluctuation (tidaling) in the water-seal compartment. 3. There is 250 mL of blood in the drainage compartment 4. The client is able to deep breathe without any pain.

***2. At three (3) days post-insertion, no fluctuation (tidaling) indicates the lung has reexpanded, which indicates the treatment has been effective. TEST-TAKING HINT: The test taker must be knowledgeable about chest tubes to be able to answer this question. The test taker must know the normal time frame and what is expected for each compartment of the chest tube drainage system.

The unlicensed nursing assistant is assisting the client with a chest tube to ambulate to the bathroom. Which situation warrants immediate intervention from the nurse? 1. The client's chest tube is below the level of the chest. 2. The nursing assistant has the chest tube attached to suction. 3. The nursing assistant allowed the client out of the bed. 4. The nursing assistant uses a bedside commode for the client.

***2. The chest tube system can function as a result of gravity and does not have to be attached to suction. Keeping it attached to suction could cause the client to trip and fall. Therefore, this is a safety issue and the nurse should intervene and explain this to the nursing assistant. TEST-TAKING HINT: "Warrants immediate intervention" means the test taker must identify the situation in which the nurse should intervene and correct the action, demonstrate a skill, or somehow intervene with the unlicensed assistant's behavior.

The alert and oriented client is diagnosed with a spontaneous pneumothorax, and the physician is preparing to insert a left-sided chest tube. Which intervention should the nurse implement first? 1. Gather the needed supplies for the procedure. 2. Obtain a signed informed consent form. 3. Assist the client into a side-lying position. 4. Discuss the procedure with the client.

***2. The insertion of a chest tube is an invasive procedure and so requires informed con- sent. Without a consent form, this procedure cannot be done on an alert and oriented client. TEST-TAKING HINT: The test taker must know that invasive procedures require informed consent and legally it must be obtained first before anyone can touch the client.

The nurse is caring for a client with a right-sided chest tube secondary to a pneu- mothorax. Which interventions should the nurse implement when caring for this client? Select all that apply. 1. Place the client in a low-Fowler's position. 2. Assess chest tube drainage system frequently. 3. Maintain strict bed rest for the client. 4. Secure a loop of drainage tubing to the sheet. 5. Observe the site for subcutaneous emphysema.

***2. The system must be patent and intact to function properly. ***4. Looping the tubing prevents direct pres- sure on the chest tube itself and keeps tubing off the floor, addressing both a safety and an infection control issue. ***5. Subcutaneous emphysema is air under the skin, which is a common occurrence at the chest tube insertion site. TEST-TAKING HINT: The test taker should be careful with adjectives. In option "1" the word "low" makes it incorrect; in option "3," the word "strict" makes this option incorrect.

The client had a right-sided chest tube inserted two (2) hours ago for a pneumothorax. Which action should the nurse take if there is no fluctuation (tidaling) in the water-seal compartment? 1. Obtain an order for a stat chest x-ray. 2. Increase the amount of wall suction. 3. Check the tubing for kinks or clots. 4. Monitor the client's pulse oximeter reading.

***3. The key to the answer is "2 hours." The air from the pleural space is not able to get to the water-seal compartment, and the nurse should try to determine why. Usually the client is lying on the tube, it is kinked, or there is a dependent loop. TEST-TAKING HINT: The test taker should apply the nursing process to answer the question correctly. The first step in the nursing process is assessment and "check" (option "3") is a word that can be used synonymously for assess. Monitoring (option "4") is also assess- ing, but the test taker should not check a diagnostic test result before caring for the client.

The nurse is presenting a class on chest tubes. Which statement describes a tension pneumothorax? 1. A tension pneumothorax develops when an air-filled bleb on the surface of the lung ruptures. 2. When a tension pneumothorax occurs, the air moves freely between the pleural space and the atmosphere. 3. The injury allows air into the pleural space but prevents it from escaping from the pleural space. 4. A tension pneumothorax results from a puncture of the pleura during a central line placement.

***3. This describes a tension pneumothorax. It is a medical emergency requiring immediate intervention to preserve life. TEST-TAKING HINT: The test taker must always be clear about what the question is asking before answering the question. If the test taker can eliminate options "1" and "2" and can't decide between "3" and "4," the test taker must go back to the stem and clarify what the question is asking.

The charge nurse is making client assignments on a medical floor. Which client should the charge nurse assign to the LPN? 1. The client with pneumonia who has a pulse oximeter reading of 91%. 2. The client with a hemothorax who has Hgb of 9 mg/dL and Hct of 20%. 3. The client with chest tubes who has jugular vein distention and BP of 96/60. 4. The client who is two (2) hours post-bronchoscopy procedure.

***4. A client that is two (2) hours post- bronchoscopy procedure could safely be assigned to an LPN. TEST-TAKING HINT: The test taker must under- stand that the LPN should be assigned the least critical client or the client that is stable and not exhibiting any complications second- ary to the admitting disease or condition.

The client has a right-sided chest tube. As the client is getting out of the bed it is acci- dentally pulled out of the pleural space. Which action should the nurse implement first? 1. Notify the health-care provider to have chest tubes reinserted STAT. 2. Instruct the client to take slow shallow breaths until the tube is reinserted. 3. Take no action and assess the client's respiratory status every 15 minutes. 4. Tape a petroleum jelly occlusive dressing on three (3) sides to the insertion site.

***4. Taping on three sides prevents the development of a tension pneumothorax by inhibiting air from entering the wound during inhalation but allowing it to escape during exhalation. TEST-TAKING HINT: The word "first interven- tion" in the stem of the question indicates to the test taker that possibly more than one (1) intervention could be indicated in the situa- tion but only one (1) is implemented first. Remember, do not select assessment first without reading the question. If the client is in any type of crisis, then the nurse should first do something to help the client's situation.

The nurse is caring for a patient who is experiencing cardiogenic shock as a result of myocardial infarction. Which of the following assessments would concern the nurse the most? 1. Pao2 60 mm/hg 2. blood pressure 100/56 mm Hg 3. urine output 260 cc/8 hrs 4. heart rate 96

1. Pao2 60 mm/hg A PaO2 of 60 is below the normal of 80 to 100 mm Hg. The patient experiencing cardiogenic shock will exhibit hypotension and tachycardia and therefore a blood pressure of 100/56 mm Hg and heart rate of 96 would not apply. A urine output of 260 cc/8 hrs is borderline but not reportable without a continued trending pattern.

A nursing student learns about modifiable risk factors for coronary artery disease. Which factors does this include? (Select all that apply.) a. Age b. Hypertension c. Obesity d. Smoking e. Stress

ANS: B, C, D, E Hypertension, obesity, smoking, and excessive stress are all modifiable risk factors for coronary artery disease. Age is a nonmodifiable risk factor.

The client comes to the emergency department with chest discomfort. Which action does the nurse perform first? 1. Administers oxygen therapy 2. Obtains the client's description of the chest discomfort 3. Provides pain relief medication 4. Remains calm and stays with the client

2. A description of the chest discomfort must be obtained before further action can be taken.

The client in the cardiac care unit has had a large myocardial infarction. How does the nurse recognize onset of left ventricular failure? 1. Urine output of 1500 mL on the preceding day 2. Crackles in the lung fields 3. Pedal edema 4. Expectoration of yellow sputum

2. Manifestations of left ventricular failure and pulmonary edema are noted by listening for crackles and identifying their locations in the lung fields. Edema is a sign of right ventricular heart failure.

The nurse is caring for a group of clients who have sustained myocardial infarction (MI). The nurse observes the client with which type of MI most carefully for the development of left ventricular heart failure? 1. Inferior wall 2. Anterior wall 3. Lateral wall 4. Posterior wall

2. Owing to the large size of the anterior wall, the amount of tissue infarction may be large enough to decrease the force of contraction, leading to heart failure. with the inferior wall, the client is more likely to develop right ventricular MI. regarding clients with obstruction of the circumflex artery may experience a lateral wall or posterior wall MI and sinus dysrhythmias.

When caring for a client with acute myocardial infarction, the nurse recognizes that prompt pain management is essential for which reason? 1.The discomfort will increase client anxiety and reduce coping. 2. Pain relief improves the oxygen supply and decreases oxygen demand. 3. Relief of pain indicates that the myocardial infarction is resolving. 4. Pain medication should not be used until a definitive diagnosis has been established.

2. The focus of pain relief is on reducing myocardial oxygen demand. Chest discomfort will increase anxiety, but it may not affect coping. The major purpose of pain relief is to reduce myocardial oxygen demand. Relief of pain is secondary to the use of opiates or indicates that the tissue infarction is complete.

Before abdominal surgery for an intestinal obstruction, the nurse monitors the client's urine output and finds that the total output for the past 2 hours was 35 mL. The nurse then assesses the client's total intake and output over the last 24 hours and notes that he had 2,000 mL of I.V. fluid for intake, 500 mL of drainage from the nasogastric tube, and 700 mL of urine for a total output of 1,200 mL. This would indicate which of the following? 1. Decreased renal function. 2. Inadequate pain relief. 3. Extension of the obstruction. 4. Inadequate fluid replacement.

4. Considering that there is usually 1 L of insensible fluid loss, this client's output exceeds his intake (intake, 2,000 mL; output, 2,200 mL), indicating deficient fluid volume. The kidneys are concentrating urine in response to low circulating volume, as evidenced by a urine output of less than 30 mL/ hour. This indicates that increased fluid replacement is needed. Decreasing urine output can be a sign of decreased renal function, but the data provided suggest that the client is dehydrated. Pain does not affect urine output. There are no data to suggest that the obstruction has worsened.

A patient has developed severe cardiogenic shock and is on a mechanical ventilator. The family asks why the patient's hands feel so cold. What is the nurse's best response? 1. "We keep the intensive care unit cool to reduce patients' metabolic rates." 2. "The patient has developed a fever and chills." 3. "This happens frequently to patients in shock states." 4. "Blood vessels constrict in shock, which takes the blood away from hands and feet."

4. "Blood vessels constrict in shock, which takes the blood away from hands and feet." Vasoconstriction results from catecholamine release, which is a compensatory mechanism in shock. The diversion of warm blood away from the extremities results in cold hands and feet.

The nurse suspects that a patient is experiencing cardiogenic shock. Which parameter indicates that the nurse's suspicion is correct? 1. Cardiac output of 8.9 L/min 2. Pulmonary artery wedge pressure (PAWP) of 8 mm Hg 3. Central venous pressure (CVP) of 5 mm Hg 4. Cardiac index (CI) of 1.8 L/min/m2

4. Cardiac index (CI) of 1.8 L/min/m2 The cardiac index (CI) is a measure of cardiac output and tissue perfusion in relation to the patient's body surface area. This reading would be consistent with cardiogenic shock.

To validate that the client has had a myocardial infarction (MI), the nurse assesses for positive findings on which tests? 1. Creatine kinase-MB fraction (CK-MB) and alkaline phosphatase 2. Homocysteine and C-reactive protein 3. Total cholesterol, low-density lipoprotein (LDL) and high-density lipoprotein (HDL) cholesterols 4. Myoglobin and troponin

4. Myoglobin, troponin, and CK-MB are the cardiac markers used to determine whether MI has occurred. Homocysteine and C-reactive protein are markers of inflammation, which may represent risk for MI, but they are not diagnostic for MI.

Which of the following associated disorders may a client with ulcerative colitis exhibit? 1. Gallstones 2. Hydronephrosis 3. Nephrolithiasis 4. Toxic megacolon

4. Toxic megacolon Toxic megacolon is extreme dilation of a segment of the diseased colon caused by paralysis of the colon, resulting in complete obstruction. This disorder is associated with both Crohn's disease and ulcerative colitis. The other disorders are more commonly associated with Crohn's disease.

Which nursing action should be implemented in the care of a patient who is experiencing increased ICP? A. Monitor fluid and electrolyte status astutely. B. Position the patient in a high-Fowler's position. C. Administer vasoconstrictors to maintain cerebral perfusion. D. Maintain physical restraints to prevent episodes of agitation.

A Fluid and electrolyte disturbances can have an adverse effect on ICP and must be vigilantly monitored. The head of the patient's bed should be kept at 30 degrees in most circumstances, and physical restraints are not applied unless absolutely necessary. Vasoconstrictors are not typically administered in the treatment of ICP.

Which pneumonia complication does the nurse recognize as creating pain that increases on inspiration because of inflammation of the parietal pleura? A. pleuritic CP B. meningitis C. COPD D. Pulmonary emboli

A Pleuritic CP

The nurse is caring for a client with an umbilical hernia who reports increased abdominal pain, nausea, and vomiting. The nurse notes high-pitched bowel sounds. Which conclusion does the nurse draw from these assessment findings? a. Bowel obstruction; client should be placed on NPO status. b. Perforation of the bowel; client needs emergency surgery. c. Adhesions in the hernia; client needs elective surgery. d. Hernia is dangerously enlarged; client needs a nasogastric (NG) tube.

A The client with a hernia presenting with abdominal pain, fever, tachycardia, nausea and vomiting, and hypoactive bowel sounds should be suspected of having developed strangulation. Strangulation poses a risk of intestinal obstruction. The client should be placed on NPO status, and the health care provider should be notified. The symptoms are not suggestive of enlargement of the hernia, adhesion formation, or bowel perforation.

A patient with a suspected closed head injury has bloody nasal drainage. You suspect that this patient has a cerebrospinal fluid (CSF) leak when observing which of the following? A. A halo sign on the nasal drip pad B. Decreased blood pressure and urinary output C. A positive reading for glucose on a Test-tape strip D. Clear nasal drainage along with the bloody discharge

A When drainage containing CSF and blood is allowed to drip onto a white pad, the blood coalesces into the center within a few minutes, and a yellowish ring of CSF encircles the blood, giving a halo effect. The presence of glucose is unreliable for determining the presence of CSF because blood also contains glucose.

The nurse is monitoring a patient for increased ICP following a head injury. Which of the following manifestations indicate an increased ICP (select all that apply) a. fever b. oriented to name only c. narrowing pulse pressure d. dilated right pupil > left pupil e. decorticate posturing to painful stimulus

A, B, D, E- The first sign of increased ICP is a change in LOC. Other manifestations are dilated ipsilateral pupil, changes in motor response such as posturing, and fever, which may indicate pressure on the hypothalamus. Changes in vital signs would be an increased systolic BP with widened pulse pressure and bradycardia

A pt is suspected on having community-acquired pneumonia. The nurse anticipates which of the following tests to be done to dx pt A. sputum gram stain B. Pulmonary function test C. fluorescein bronchoscopy D. peak flow meter measurement E. chest x-ay

A, E sputum gram stain CXR

The complex care provided during an emergency requires interdisciplinary collaboration. Which interdisciplinary team members are paired with the correct responsibilities? (SATA) a. Psychiatric crisis nurse Interacts with clients and families when sudden illness, serious injury, or death of a loved one may cause a crisis b. Forensic nurse examiner Performs rapid assessments to ensure clients with the highest acuity receive the quickest evaluation, treatment, and prioritization of resources c. Triage nurse Provides basic life support interventions such as oxygen, basic wound care, splinting, spinal immobilization, and monitoring of vital signs d. Emergency medical technician Obtains client histories, collects evidence, and offers counseling and follow-up care for victims of rape, child abuse, and domestic violence e. Paramedic Provides prehospital advanced life support, including cardiac monitoring, advanced airway management, and medication administration

A, E ~ The psychiatric crisis nurse evaluates clients with emotional behaviors or mental illness and facilitates follow-up treatment plans. The psychiatric crisis nurse also works with clients and families when experiencing a crisis. Paramedics are advanced life support providers who can perform advanced techniques that may include cardiac monitoring, advanced airway management and intubation, establishing IV access, and administering drugs en route to the emergency department. The forensic nurse examiner is trained to recognize evidence of abuse and to intervene on the clients behalf. The forensic nurse examiner will obtain client histories, collect evidence, and offer counseling and follow-up care for victims of rape, child abuse, and domestic violence. The triage nurse performs rapid assessments to ensure clients with the highest acuity receive the quickest evaluation, treatment, and prioritization of resources. The emergency medical technician is usually the first caregiver and provides basic life support and transportation to the emergency department.

You are providing care for a patient who has been admitted to the hospital with a head injury who requires regular neurologic vital signs. Which assessments are components of the patient's score on the Glasgow Coma Scale (select all that apply)? A. Eye opening B. Abstract reasoning C. Best verbal response D. Best motor response E. Cranial nerve function

A,C,D The three dimensions of the Glasgow Coma Scale are eye opening, best verbal response, and best motor response.

A 7 year old client is brought to the E.R. He's tachypnec and afebrile and has a RR of 36 bpm, and a nonproductive cough. He recently had a cold. From his history, the client may have which of the following? A. Acute asthma B. Bronchial pneumonia C. Chronic obstructive pulmonary disease (COPD) D. Emphysema

A. Acute asthma Based on the client's history & symptoms, acute asthma is the most likely diagnosis. He's unlikely to have bronchial pneumonia without a productive cough & fever. He is too young to have developed COPD or emphysema.

An elderly client with pneumonia may appear with which of the following symptoms first? A. Altered mental status & dehydration B. Fever & chills C. Hemoptysis & dyspnea D. Pleuritic chest pain & cough

A. Altered mental status & dehydration Elderly clients may first appear with only an altered mental status & dehydration due to a blunted immune response.

A patient has ICP monitoring with an intraventricular catheter. A priority nursing intervention for the patient is a. aseptic technique to prevent infection b. constant monitoring of ICP waveforms c. removal of CSF to maintain normal ICP d. sampling CSF to determine abnormalities

A. Aseptic technique to prevent infection- An intraventricular catheter is a fluid coupled system that can provide direct access for microorganisms to enter the ventricles of the brain, and aseptic technique is a very high nursing priority to decrease the risk for infection. Constant monitoring of ICP waveforms is not usually necessary, and removal of CSF for sampling or to maintain normal ICP is done only when specifically ordered

Which of the following respiratory disorders is the most common in the first 24-48 hours after surgery? A. Atelectasis B. Bronchitis C. Pneumonia D. Pneumothorax

A. Atelectasis Atelectasis develops when there's interference with the normal negative pressure that promotes lung expansion. Clients in the postoperative phase often splint their breathing because of pain & positioning, which causes hypoxia.

A female patient who had a stroke 24 hours ago has expressive aphasia. The nurse identifies the nursing diagnosis of impaired verbal communication. An appropriate nursing intervention to help the patient communicate is to a. ask questions that the patient can answer with "yes" or "no." b. develop a list of words that the patient can read and practice reciting. c. have the patient practice her facial and tongue exercises with a mirror. d. prevent embarrassing the patient by answering for her if she does not respond.

A. Communication will be facilitated and less frustrating to the patient when questions that require a "yes" or "no" response are used. When the language areas of the brain are injured, the patient might not be able to read or recite words, which will frustrate the patient without improving communication. Expressive aphasia is caused by damage to the language areas of the brain, not by the areas that control the motor aspects of speech. The nurse should allow time for the patient to respond.

Which signs and symptoms differentiate hypoxemic respiratory failure from hypercapnic respiratory failure (select all that apply)? A. Cyanosis B. Tachypnea C. Morning headache D. Paradoxic breathing E. Pursed-lip breathing

A. Cyanosis B. Tachypnea D. Paradoxic breathing Clinical manifestations that occur with hypoxemic respiratory failure include cyanosis, tachypnea, and paradoxic chest or abdominal wall movement with the respiratory cycle. Clinical manifestations of hypercapnic respiratory failure include morning headache, pursed-lip breathing, and decreased or increase respiratory rate with shallow breathing.

What pathophysiologic condition can result in ARDS? A. Damage to the alveolar-capillary membrane B. Copious exudates production C. Airway spasms and vasoconstriction D. Change in the inspiratory-to-expiratory ratio

A. Damage to the alveolar-capillary membrane In ARDS, there is damage to the alveolar-capillary membrane, although the exact mechanism is not known. The damage results in increased pulmonary capillary membrane permeability, destruction of elastic and collagen, formation of pulmonary microemboli, and pulmonary artery vasoconstriction. These changes produce increased fluid accumulation and decreased lung compliance. Temporary narrowing of the airway is seen in asthma. Exudate production is seen with pneumonia or chronic obstructive pulmonary disease (COPD). The cause does not involve a change in ventilation, although there may eventually be some alteration due to respiratory distress.

Which is a classic finding for a patient with ARDS? A. Hypoxemia despite increased oxygen administration B. Bronchodilators ordered to relieve airway spasms C. Development of Kussmaul respirations D. Development of Cheyne-Stokes respirations

A. Hypoxemia despite increased oxygen administration The hallmark of ARDS is hypoxemia despite increased FIO2 by mask, cannula, or endotracheal tube. Bronchodilators are used for asthma. Kussmaul respirations are caused by metabolic acidosis in diabetic ketoacidosis. Cheyne-Stokes respirations are a stairstep respiratory pattern with periods of apnea related to the body being stimulated by high CO2 levels to breathe. It is seen in patients with increased intracranial pressure.

When a patient is admitted to the emergency department following a head injury, the nurse's first priority in management of the patient once a patent airway is confirmed is a. maintaining cervical spine precautions b. determining the presence of increased ICP c. monitoring for changes in neurologic status d. establishing IV access with a large-bore catheter

A. In addition to monitoring for a patent airway during emergency care of the patient with a head injury, the nurse must always assume that a patient with a head injury may have a cervical spine injury. Maintaining cervical spine precautions in all assessment and treatment activities with the patient is essential to prevent additional neurologic damage.

Emergency treatment of a client in status asthmaticus includes which of the following meds? A. Inhaled beta-adrenergic agents B. Inhaled corticosterioids C. I.V. beta-adrenergic agents D. Oral corticosterioids

A. Inhaled beta-adrenergic agents Inhaled beta-adrenergic agents help promote bronchodilation, which improves oxygenation. I.V. beta-adrenergic agents can be used but they have to be monitored for their greater systemic effects

During admission of a patient with a severe head injury to the ED, the nurse places highest priority on assessment for a. patency of of airway b. presence of a neck injury c. neurologic status with Glascow Coma Scale d. CSF leakage from ears and nose

A. Patency of airway is the #1 priority with all head injuries

A patient with a severe acute asthma exacerbation presents to the emergency department. Over the next hour, the patient remains in respiratory distress, but the respirations have slowed. What is the best explanation? A. The patient is developing respiratory muscle fatigue. B. The respirations are exchanging oxygen and carbon dioxide more efficiently. C. The patient's anxiety level is lessening. D. The body has compensated by retaining sodium bicarbonate.

A. The patient is developing respiratory muscle fatigue. A rapid respiratory rate requires a substantial amount of work. Change from a rapid rate to a slower rate in a patient in acute respiratory distress suggests extreme progression of respiratory muscle fatigue and increased probability of respiratory arrest. Ventilatory exchange, without other indications of improvement, is decreased. As long as the patient is in distress, there is no evidence that anxiety would lessen, and hypoxia would increase anxiety. Compensation through the renal system takes days.

In which order will the nurse take these actions when assisting with oral intubation of a patient who is having respiratory distress? Put a comma and space between each answer choice (a, b, c, d, etc.) ____________________ a. Obtain a portable chest-x-ray. b. Place the patient in the supine position. c. Inflate the cuff of the endotracheal tube. d. Attach an end-tidal CO2 detector to the endotracheal tube. e. Oxygenate the patient with a bag-valve-mask system for several minutes.

ANS: E, B, C, D, A The patient is pre-oxygenated with a bag-valve-mask system for 3 to 5 minutes before intubation and then placed in a supine position. Following the intubation, the cuff on the endotracheal tube is inflated to occlude and protect the airway. Tube placement is assessed first with an end-tidal CO2 sensor, then with a chest x-ray. DIF: Cognitive Level: Analysis REF: 1701-1702 OBJ: Special Questions: Alternate Item Format, Prioritization TOP: Nursing Process: Implementation MSC: NCLEX: Physiological Integrity

A client in the emergency department has several broken ribs. What care measure will best promote comfort? a. Allowing the client to choose the position in bed b. Humidifying the supplemental oxygen c. Offering frequent, small drinks of water d. Providing warmed blankets

ANS: A Allow the client with respiratory problems to assume a position of comfort if it does not interfere with care. Often the client will choose a more upright position, which also improves oxygenation. The other options are less effective comfort measures.

A nurse assesses clients on a cardiac unit. Which client should the nurse identify as being at greatest risk for the development of left-sided heart failure? a. A 36-year-old woman with aortic stenosis b. A 42-year-old man with pulmonary hypertension c. A 59-year-old woman who smokes cigarettes daily d. A 70-year-old man who had a cerebral vascular accident

ANS: A Although most people with heart failure will have failure that progresses from left to right, it is possible to have left-sided failure alone for a short period. It is also possible to have heart failure that progresses from right to left. Causes of left ventricular failure include mitral or aortic valve disease, coronary artery disease, and hypertension. Pulmonary hypertension and chronic cigarette smoking are risk factors for right ventricular failure. A cerebral vascular accident does not increase the risk of heart failure.

A client had an embolic stroke and is having an echocardiogram. When the client asks why the provider ordered "a test on my heart," how should the nurse respond? a. "Most of these types of blood clots come from the heart." b. "Some of the blood clots may have gone to your heart too." c. "We need to see if your heart is strong enough for therapy." d. "Your heart may have been damaged in the stroke too."

ANS: A An embolic stroke is caused when blood clots travel from one area of the body to the brain. The most common source of the clots is the heart. The other statements are inaccurate.

A nurse is teaching a client with heart failure who has been prescribed enalapril (Vasotec). Which statement should the nurse include in this client's teaching? a. "Avoid using salt substitutes." b. "Take your medication with food." c. "Avoid using aspirin-containing products." d. "Check your pulse daily."

ANS: A Angiotensin-converting enzyme (ACE) inhibitors such as enalapril inhibit the excretion of potassium. Hyperkalemia can be a life-threatening side effect, and clients should be taught to limit potassium intake. Salt substitutes are composed of potassium chloride. ACE inhibitors do not need to be taken with food and have no impact on the client's pulse rate. Aspirin is often prescribed in conjunction with ACE inhibitors and is not contraindicated.

A nurse admits a client who is experiencing an exacerbation of heart failure. Which action should the nurse take first? a. Assess the client's respiratory status. b. Draw blood to assess the client's serum electrolytes. c. Administer intravenous furosemide (Lasix). d. Ask the client about current medications.

ANS: A Assessment of respiratory and oxygenation status is the priority nursing intervention for the prevention of complications. Monitoring electrolytes, administering diuretics, and asking about current medications are important but do not take priority over assessing respiratory status.

A client has a brain abscess and is receiving phenytoin (Dilantin). The spouse questions the use of the drug, saying the client does not have a seizure disorder. What response by the nurse is best? a. "Increased pressure from the abscess can cause seizures." b. "Preventing febrile seizures with an abscess is important." c. "Seizures always occur in clients with brain abscesses." d. "This drug is used to sedate the client with an abscess."

ANS: A Brain abscesses can lead to seizures as a complication. The nurse should explain this to the spouse. Phenytoin is not used to prevent febrile seizures. Seizures are possible but do not always occur in clients with brain abscesses. This drug is not used for sedation.

A nursing student is caring for a client who had a myocardial infarction. The student is confused because the client states nothing is wrong and yet listens attentively while the student provides education on lifestyle changes and healthy menu choices. What response by the faculty member is best? a. "Continue to educate the client on possible healthy changes." b. "Emphasize complications that can occur with noncompliance." c. "Tell the client that denial is normal and will soon go away." d. "You need to make sure the client understands this illness."

ANS: A Clients are often in denial after a coronary event. The client who seems to be in denial but is compliant with treatment may be using a healthy form of coping that allows time to process the event and start to use problem-focused coping. The student should not discourage this type of denial and coping, but rather continue providing education in a positive manner. Emphasizing complications may make the client defensive and more anxious. Telling the client that denial is normal is placing too much attention on the process. Forcing the client to verbalize understanding of the illness is also potentially threatening to the client.

A nurse cares for a client with right-sided heart failure. The client asks, "Why do I need to weigh myself every day?" How should the nurse respond? a. "Weight is the best indication that you are gaining or losing fluid." b. "Daily weights will help us make sure that you're eating properly." c. "The hospital requires that all inpatients be weighed daily." d. "You need to lose weight to decrease the incidence of heart failure."

ANS: A Daily weights are needed to document fluid retention or fluid loss. One liter of fluid equals 2.2 pounds. The other responses do not address the importance of monitoring fluid retention or loss.

A client with a stroke has damage to Broca's area. What intervention to promote communication is best for this client? a. Assess whether or not the client can write. b. Communicate using "yes-or-no" questions. c. Reinforce speech therapy exercises. d. Remind the client not to use neologisms.

ANS: A Damage to Broca's area often leads to expressive aphasia, wherein the client can understand what is said but cannot express thoughts verbally. In some instances the client can write. The nurse should assess to see if that ability is intact. "Yes-or-no" questions are not good for this type of client because he or she will often answer automatically but incorrectly. Reinforcing speech therapy exercises is good for all clients with communication difficulties. Neologisms are made-up "words" often used by clients with sensory aphasia.

A client is on a dopamine infusion via a peripheral line. What action by the nurse takes priority for safety? a. Assess the IV site hourly. b. Monitor the pedal pulses. c. Monitor the client's vital signs. d. Obtain consent for a central line.

ANS: A Dopamine should be infused through a central line to prevent extravasation and necrosis of tissue. If it needs to be run peripherally, the nurse assesses the site hourly for problems. When the client is getting the central line, ensuring informed consent is on the chart is a priority. But at this point, the client has only a peripheral line, so caution must be taken to preserve the integrity of the client's integumentary system. Monitoring pedal pulses and vital signs give indications as to how well the drug is working.

A nurse evaluates a client with acute glomerulonephritis (GN). Which manifestation should the nurse recognize as a positive response to the prescribed treatment? a. The client has lost 11 pounds in the past 10 days. b. The client's urine specific gravity is 1.048. c. No blood is observed in the client's urine. d. The client's blood pressure is 152/88 mm Hg.

ANS: A Fluid retention is a major feature of acute GN. This weight loss represents fluid loss, indicating that the glomeruli are performing the function of filtration. A urine specific gravity of 1.048 is high. Blood is not usually seen in GN, so this finding would be expected. A blood pressure of 152/88 mm Hg is too high; this may indicate kidney damage or fluid overload.

A nurse is providing community screening for risk factors associated with stroke. Which client would the nurse identify as being at highest risk for a stroke? a. A 27-year-old heavy cocaine user b. A 30-year-old who drinks a beer a day c. A 40-year-old who uses seasonal antihistamines d. A 65-year-old who is active and on no medications

ANS: A Heavy drug use, particularly cocaine, is a risk factor for stroke. Heavy alcohol use is also a risk factor, but one beer a day is not considered heavy drinking. Antihistamines may contain phenylpropanolamine, which also increases the risk for stroke, but this client uses them seasonally and there is no information that they are abused or used heavily. The 65-year-old has only age as a risk factor.

A client in the intensive care unit is started on continuous venovenous hemofiltration (CVVH). Which finding is the cause of immediate action by the nurse? a. Blood pressure of 76/58 mm Hg b. Sodium level of 138 mEq/L c. Potassium level of 5.5 mEq/L d. Pulse rate of 90 beats/min

ANS: A Hypotension can be a problem with CVVH if replacement fluid does not provide enough volume to maintain blood pressure. The specially trained nurse needs to monitor for ongoing fluid and electrolyte replacement. The sodium level is normal and the potassium level is slightly elevated, which could be normal findings for someone with acute kidney injury. A pulse rate of 90 beats/min is normal.

A nurse cares for a client who has obstructive jaundice. The client asks, "Why is my skin so itchy?" How should the nurse respond? a. "Bile salts accumulate in the skin and cause the itching." b. "Toxins released from an inflamed gallbladder lead to itching." c. "Itching is caused by the release of calcium into the skin." d. "Itching is caused by a hypersensitivity reaction."

ANS: A In obstructive jaundice, the normal flow of bile into the duodenum is blocked, allowing excess bile salts to accumulate on the skin. This leads to itching, or pruritus. The other statements are not accurate.

An older adult is on cardiac monitoring after a myocardial infarction. The client shows frequent dysrhythmias. What action by the nurse is most appropriate? a. Assess for any hemodynamic effects of the rhythm. b. Prepare to administer antidysrhythmic medication. c. Notify the provider or call the Rapid Response Team. d. Turn the alarms off on the cardiac monitor.

ANS: A Older clients may have dysrhythmias due to age-related changes in the cardiac conduction system. They may have no significant hemodynamic effects from these changes. The nurse should first assess for the effects of the dysrhythmia before proceeding further. The alarms on a cardiac monitor should never be shut off. The other two actions may or may not be needed.

The nurse cares for a patient who has just had a thoracentesis. Which assessment information obtained by the nurse is a priority to communicate to the health care provider? a. Oxygen saturation is 88%. b. Blood pressure is 145/90 mm Hg. c. Respiratory rate is 22 breaths/minute when lying flat. d. Pain level is 5 (on 0 to 10 scale) with a deep breath.

ANS: A Oxygen saturation would be expected to improve after a thoracentesis. A saturation of 88% indicates that a complication such as pneumothorax may be occurring. The other assessment data also indicate a need for ongoing assessment or intervention, but the low oxygen saturation is the priority.

A patient experiences a chest wall contusion as a result of being struck in the chest with a baseball bat. The emergency department nurse would be most concerned if which finding is observed during the initial assessment? a. Paradoxic chest movement b. Complaint of chest wall pain c. Heart rate of 110 beats/minute d. Large bruised area on the chest

ANS: A Paradoxic chest movement indicates that the patient may have flail chest, which can severely compromise gas exchange and can rapidly lead to hypoxemia. Chest wall pain, a slightly elevated pulse rate, and chest bruising all require further assessment or intervention, but the priority concern is poor gas exchange.

A client is in the clinic for a follow-up visit after a moderate traumatic brain injury. The client's spouse is very frustrated, stating that the client's personality has changed and the situation is intolerable. What action by the nurse is best? a. Explain that personality changes are common following brain injuries. b. Ask the client why he or she is acting out and behaving differently. c. Refer the client and spouse to a head injury support group. d. Tell the spouse this is expected and he or she will have to learn to cope.

ANS: A Personality and behavior often change permanently after head injury. The nurse should explain this to the spouse. Asking the client about his or her behavior isn't useful because the client probably cannot help it. A referral might be a good idea, but the nurse needs to do something in addition to just referring the couple. Telling the spouse to learn to cope belittles the spouse's concerns and feelings.

A client has a shoulder injury and is scheduled for a magnetic resonance imaging (MRI). The nurse notes the presence of an aneurysm clip in the client's record. What action by the nurse is best? a. Ask the client how long ago the clip was placed. b. Have the client sign an informed consent form. c. Inform the provider about the aneurysm clip. d. Reschedule the client for computed tomography.

ANS: A Some older clips are metal, which would preclude the use of MRI. The nurse should determine how old the clip is and relay that information to the MRI staff. They can determine if the client is a suitable candidate for this examination. The client does not need to sign informed consent. The provider will most likely not know if the client can have an MRI with this clip. The nurse does not independently change the type of diagnostic testing the client receives.

An elderly patient who has stabilized after being in the intensive care unit (ICU) for a week is preparing for transfer to the step down unit when the nurse notices that the patient has new onset confusion. The nurse will plan to a. inform the receiving nurse and then transfer the patient. b. notify the health care provider and postpone the transfer. c. administer PRN lorazepam (Ativan) and cancel the transfer. d. obtain an order for restraints as needed and transfer the patient.

ANS: A The patient's history and symptoms most likely indicate delirium associated with the sleep deprivation and sensory overload in the ICU environment, and informing the receiving nurse and transferring the patient is appropriate. Postponing the transfer is likely to prolong the delirium. Benzodiazepines and restraints contribute to delirium and agitation. DIF: Cognitive Level: Application REF: 1686 TOP: Nursing Process: Planning MSC: NCLEX: Psychosocial Integrity

A nurse cares for a client with hepatopulmonary syndrome who is experiencing dyspnea with oxygen saturations at 92%. The client states, "I do not want to wear the oxygen because it causes my nose to bleed. Get out of my room and leave me alone!" Which action should the nurse take? a. Instruct the client to sit in as upright a position as possible. b. Add humidity to the oxygen and encourage the client to wear it. c. Document the client's refusal, and call the health care provider. d. Contact the provider to request an extra dose of the client's diuretic.

ANS: A The client with hepatopulmonary syndrome is often dyspneic. Because the oxygen saturation is not significantly low, the nurse should first allow the client to sit upright to see if that helps. If the client remains dyspneic, or if the oxygen saturation drops further, the nurse should investigate adding humidity to the oxygen and seeing whether the client will tolerate that. The other two options may be beneficial, but they are not the best choices. If the client is comfortable, his or her agitation will decrease; this will improve respiratory status.

A patient with respiratory failure has hemodynamic monitoring and is receiving mechanical ventilation with peak end-expiratory pressure (PEEP) of 10 cm H2O. Which information indicates that a change in the ventilator settings may be required? a. The arterial line shows a blood pressure of 90/46. b. The pulmonary artery pressure (PAP) is decreased. c. The cardiac monitor shows a heart rate of 58 beats/min. d. The pulmonary artery wedge pressure (PAWP) is increased.

ANS: A The hypotension indicates that the high intrathoracic pressure caused by the PEEP may be decreasing venous return and cardiac output (CO). The other assessment data would not be caused by mechanical ventilation. DIF: Cognitive Level: Application REF: 1710 TOP: Nursing Process: Evaluation MSC: NCLEX: Physiological Integrity

Which assessment information obtained by the nurse when caring for a patient receiving mechanical ventilation indicates the need for suctioning? a. The respiratory rate is 32 breaths/min. b. The pulse oximeter shows a SpO2 of 93%. c. The patient has not been suctioned for the last 6 hours. d. The lungs have occasional audible expiratory wheezes.

ANS: A The increase in respiratory rate indicates that the patient may have decreased airway clearance and requires suctioning. Suctioning is done when patient assessment data indicate that it is needed, not on a scheduled basis. Occasional expiratory wheezes do not indicate poor airway clearance, and suctioning the patient may induce bronchospasm and increase wheezing. An SpO2 of 93% is acceptable and does not suggest that immediate suctioning is needed. DIF: Cognitive Level: Application REF: 1702-1704 TOP: Nursing Process: Assessment MSC: NCLEX: Physiological Integrity

A nurse is caring for a client on mechanical ventilation and finds the client agitated and thrashing about. What action by the nurse is most appropriate? a. Assess the cause of the agitation. b. Reassure the client that he or she is safe. c. Restrain the client's hands. d. Sedate the client immediately.

ANS: A The nurse needs to determine the cause of the agitation. The inability to communicate often makes clients anxious, even to the point of panic. Pain and confusion can also cause agitation. Once the nurse determines the cause of the agitation, he or she can implement measures to relieve the underlying cause. Reassurance is also important but may not address the etiology of the agitation. Restraints and more sedation may be necessary, but not as a first step.

A client in the emergency department is having a stroke and needs a carotid artery angioplasty with stenting. The client's mental status is deteriorating. What action by the nurse is most appropriate? a. Attempt to find the family to sign a consent. b. Inform the provider that the procedure cannot occur. c. Nothing; no consent is needed in an emergency. d. Sign the consent form for the client.

ANS: A The nurse should attempt to find the family to give consent. If no family is present or can be found, under the principle of emergency consent, a life-saving procedure can be performed without formal consent. The nurse should not just sign the consent form.

The nurse notes that a patient's endotracheal tube (ET), which was at the 21-cm mark, is now at the 24-cm mark and the patient appears anxious and restless. Which action should the nurse take first? a. Listen to the patient's lungs. b. Offer reassurance to the patient. c. Bag the patient at an FIO2 of 100%. d. Notify the patient's health care provider.

ANS: A The nurse should first determine whether the ET tube has been displaced into the right mainstem bronchus by listening for unilateral breath sounds. If so, assistance will be needed to reposition the tube immediately. The other actions also are appropriate, but detection and correction of tube malposition are the most critical actions. DIF: Cognitive Level: Application REF: 1701-1702 OBJ: Special Questions: Prioritization TOP: Nursing Process: Implementation MSC: NCLEX: Physiological Integrity

While assessing a client on a cardiac unit, a nurse identifies the presence of an S3 gallop. Which action should the nurse take next? a. Assess for symptoms of left-sided heart failure. b. Document this as a normal finding. c. Call the health care provider immediately. d. Transfer the client to the intensive care unit.

ANS: A The presence of an S3 gallop is an early diastolic filling sound indicative of increasing left ventricular pressure and left ventricular failure. The other actions are not warranted.

A client has a serum potassium level of 6.5 mmol/L, a serum creatinine level of 2 mg/dL, and a urine output of 350 mL/day. What is the best action by the nurse? a. Place the client on a cardiac monitor immediately. b. Teach the client to limit high-potassium foods. c. Continue to monitor the client's intake and output. d. Ask to have the laboratory redraw the blood specimen.

ANS: A The priority action by the nurse should be to check the cardiac status with a monitor. High potassium levels can lead to dysrhythmias. The other choices are logical nursing interventions for acute kidney injury but not the best immediate action.

A nurse cares for a client who is prescribed lactulose (Heptalac). The client states, "I do not want to take this medication because it causes diarrhea." How should the nurse respond? a. "Diarrhea is expected; that's how your body gets rid of ammonia." b. "You may take Kaopectate liquid daily for loose stools." c. "Do not take any more of the medication until your stools firm up." d. "We will need to send a stool specimen to the laboratory."

ANS: A The purpose of administering lactulose to this client is to help ammonia leave the circulatory system through the colon. Lactulose draws water into the bowel with its high osmotic gradient, thereby producing a laxative effect and subsequently evacuating ammonia from the bowel. The client must understand that this is an expected and therapeutic effect for him or her to remain compliant. The nurse should not suggest administering anything that would decrease the excretion of ammonia or holding the medication. There is no need to send a stool specimen to the laboratory because diarrhea is the therapeutic response to this medication.

A male client comes into the emergency department with a serum creatinine of 2.2 mg/dL and a blood urea nitrogen (BUN) of 24 mL/dL. What question should the nurse ask first when taking this client's history? a. "Have you been taking any aspirin, ibuprofen, or naproxen recently?" b. "Do you have anyone in your family with renal failure?" c. "Have you had a diet that is low in protein recently?" d. "Has a relative had a kidney transplant lately?"

ANS: A There are some medications that are nephrotoxic, such as the nonsteroidal anti-inflammatory drugs ibuprofen, aspirin, and naproxen. This would be a good question to initially ask the client since both the serum creatinine and BUN are elevated, indicating some renal problems. A family history of renal failure and kidney transplantation would not be part of the questioning and could cause anxiety in the client. A diet high in protein could be a factor in an increased BUN.

A client has a traumatic brain injury. The nurse assesses the following: pulse change from 82 to 60 beats/min, pulse pressure increase from 26 to 40 mm Hg, and respiratory irregularities. What action by the nurse takes priority? a. Call the provider or Rapid Response Team. b. Increase the rate of the IV fluid administration. c. Notify respiratory therapy for a breathing treatment. d. Prepare to give IV pain medication.

ANS: A These manifestations indicate Cushing's syndrome, a potentially life-threatening increase in intracranial pressure (ICP), which is an emergency. Immediate medical attention is necessary, so the nurse notifies the provider or the Rapid Response Team. Increasing fluids would increase the ICP. The client does not need a breathing treatment or pain medication.

A marathon runner comes into the clinic and states "I have not urinated very much in the last few days." The nurse notes a heart rate of 110 beats/min and a blood pressure of 86/58 mm Hg. Which action by the nurse is the priority? a. Give the client a bottle of water immediately. b. Start an intravenous line for fluids. c. Teach the client to drink 2 to 3 liters of water daily. d. Perform an electrocardiogram.

ANS: A This athlete is mildly dehydrated as evidenced by the higher heart rate and lower blood pressure. The nurse can start hydrating the client with a bottle of water first, followed by teaching the client to drink 2 to 3 liters of water each day. An intravenous line may be ordered later, after the client's degree of dehydration is assessed. An electrocardiogram is not necessary at this time.

A client who had a severe traumatic brain injury is being discharged home, where the spouse will be a full-time caregiver. What statement by the spouse would lead the nurse to provide further education on home care? a. "I know I can take care of all these needs by myself." b. "I need to seek counseling because I am very angry." c. "Hopefully things will improve gradually over time." d. "With respite care and support, I think I can do this."

ANS: A This caregiver has unrealistic expectations about being able to do everything without help. Acknowledging anger and seeking counseling show a realistic outlook and plans for accomplishing goals. Hoping for improvement over time is also realistic, especially with the inclusion of the word "hopefully." Realizing the importance of respite care and support also is a realistic outlook.

A nurse assesses a client with the National Institutes of Health (NIH) Stroke Scale and determines the client's score to be 36. How should the nurse plan care for this client? a. The client will need near-total care. b. The client will need cuing only. c. The client will need safety precautions. d. The client will be discharged home.

ANS: A This client has severe neurologic deficits and will need near-total care. Safety precautions are important but do not give a full picture of the client's dependence. The client will need more than cuing to complete tasks. A home discharge may be possible, but this does not help the nurse plan care for a very dependent client.

A client has a subarachnoid bolt. What action by the nurse is most important? a. Balancing and recalibrating the device b. Documenting intracranial pressure readings c. Handling the fiberoptic cable with care to avoid breakage d. Monitoring the client's phlebostatic axis

ANS: A This device needs frequent balancing and recalibration in order to read correctly. Documenting readings is important, but it is more important to ensure the device's accuracy. The fiberoptic transducer-tipped catheter has a cable that must be handled carefully to avoid breaking it, but ensuring the device's accuracy is most important. The phlebostatic axis is not related to neurologic monitoring.

The nurse is caring for a patient who has a right-sided chest tube after a right lower lobectomy. Which nursing action can the nurse delegate to the unlicensed assistive personnel (UAP)? a. Document the amount of drainage every eight hours. b. Obtain samples of drainage for culture from the system. c. Assess patient pain level associated with the chest tube. d. Check the water-seal chamber for the correct fluid level.

ANS: A UAP education includes documentation of intake and output. The other actions are within the scope of practice and education of licensed nursing personnel.

A nurse is seeing many clients in the neurosurgical clinic. With which clients should the nurse plan to do more teaching? (Select all that apply.) a. Client with an aneurysm coil placed 2 months ago who is taking ibuprofen (Motrin) for sinus headaches b. Client with an aneurysm clip who states that his family is happy there is no chance of recurrence c. Client who had a coil procedure who says that there will be no problem following up for 1 year d. Client who underwent a flow diversion procedure 3 months ago who is taking docusate sodium (Colace) for constipation e. Client who underwent surgical aneurysm ligation 3 months ago who is planning to take a Caribbean cruise

ANS: A, B After a coil procedure, up to 20% of clients experience re-bleeding in the first year. The client with this coil should not be taking drugs that interfere with clotting. An aneurysm clip can move up to 5 years after placement, so this client and family need to be watchful for changing neurologic status. The other statements show good understanding.

A client has a small-bore feeding tube (Dobhoff tube) inserted for continuous enteral feedings while recovering from a traumatic brain injury. What actions should the nurse include in the client's care? (Select all that apply.) a. Assess tube placement per agency policy. b. Keep the head of the bed elevated at least 30 degrees. c. Listen to lung sounds at least every 4 hours. d. Run continuous feedings on a feeding pump. e. Use blue dye to determine proper placement.

ANS: A, B, C, D All of these options are important for client safety when continuous enteral feedings are in use. Blue dye is not used because it can cause lung injury if aspirated.

The nurse caring for mechanically ventilated clients uses best practices to prevent ventilator-associated pneumonia. What actions are included in this practice? (Select all that apply.) a. Adherence to proper hand hygiene b. Administering anti-ulcer medication c. Elevating the head of the bed d. Providing oral care per protocol e. Suctioning the client on a regular schedule

ANS: A, B, C, D The "ventilator bundle" is a group of care measures to prevent ventilator-associated pneumonia. Actions in the bundle include using proper hand hygiene, giving anti-ulcer medications, elevating the head of the bed, providing frequent oral care per policy, preventing aspiration, and providing pulmonary hygiene measures. Suctioning is done as needed.

The nurse caring for mechanically ventilated clients knows that older adults are at higher risk for weaning failure. What age-related changes contribute to this? (Select all that apply.) a. Chest wall stiffness b. Decreased muscle strength c. Inability to cooperate d. Less lung elasticity e. Poor vision and hearing

ANS: A, B, D Age-related changes that increase the difficulty of weaning older adults from mechanical ventilation include increased stiffness of the chest wall, decreased muscle strength, and less elasticity of lung tissue. Not all older adults have an inability to cooperate or poor sensory acuity.

A client is undergoing hemodialysis. The client's blood pressure at the beginning of the procedure was 136/88 mm Hg, and now it is 110/54 mm Hg. What actions should the nurse perform to maintain blood pressure? (Select all that apply.) a. Adjust the rate of extracorporeal blood flow. b. Place the client in the Trendelenburg position. c. Stop the hemodialysis treatment. d. Administer a 250-mL bolus of normal saline. e. Contact the health care provider for orders.

ANS: A, B, D Hypotension occurs often during hemodialysis treatments as a result of vasodilation from the warmed dialysate. Modest decreases in blood pressure, as is the case with this client, can be maintained with rate adjustment, Trendelenburg positioning, and a fluid bolus. If the blood pressure drops considerably after two boluses and cooling dialysate, the hemodialysis can be stopped and the health care provider contacted.

A nurse assesses a client with nephrotic syndrome. For which clinical manifestations should the nurse assess? (Select all that apply.) a. Proteinuria b. Hypoalbuminemia c. Dehydration d. Lipiduria e. Dysuria f. Costovertebral angle (CV A) tenderness

ANS: A, B, D Nephrotic syndrome is caused by glomerular damage and is characterized by proteinuria (protein level higher than 3.5 g/24 hr), hypoalbuminemia, edema, and lipiduria. Fluid overload leading to edema and hypertension is common with nephrotic syndrome; dehydration does not occur. Dysuria is present with cystitis. CVA tenderness is present with inflammatory changes in the kidney.

A client is unsure of the decision to undergo peritoneal dialysis (PD) and wishes to discuss the advantages of this treatment with the nurse. Which statements by the nurse are accurate regarding PD? (Select all that apply.) a. "You will not need vascular access to perform PD." b. "There is less restriction of protein and fluids." c. "You will have no risk for infection with PD." d. "You have flexible scheduling for the exchanges." e. "It takes less time than hemodialysis treatments."

ANS: A, B, D PD is based on exchanges of waste, fluid, and electrolytes in the peritoneal cavity. There is no need for vascular access. Protein is lost in the exchange, which allows for more protein and fluid in the diet. There is flexibility in the time for exchanges, but the treatment takes a longer period of time compared to hemodialysis. There still is risk for infection with PD, especially peritonitis.

A nurse prepares to discharge a client who has heart failure. Based on the Heart Failure Core Measure Set, which actions should the nurse complete prior to discharging this client? (Select all that apply.) a. Teach the client about dietary restrictions. b. Ensure the client is prescribed an angiotensin-converting enzyme (ACE) inhibitor. c. Encourage the client to take a baby aspirin each day. d. Confirm that an echocardiogram has been completed. e. Consult a social worker for additional resources.

ANS: A, B, D The Heart Failure Core Measure Set includes discharge instructions on diet, activity, medications, weight monitoring and plan for worsening symptoms, evaluation of left ventricular systolic function (usually with an echocardiogram), and prescribing an ACE inhibitor or angiotensin receptor blocker. Aspirin is not part of the Heart Failure Core Measure Set and is usually prescribed for clients who experience a myocardial infarction. Although the nurse may consult the social worker or case manager for additional resources, this is not part of the Core Measures.

A nurse is working with many stroke clients. Which clients would the nurse consider referring to a mental health provider on discharge? (Select all that apply.) a. Client who exhibits extreme emotional lability b. Client with an initial National Institutes of Health (NIH) Stroke Scale score of 38 c. Client with mild forgetfulness and a slight limp d. Client who has a past hospitalization for a suicide attempt e. Client who is unable to walk or eat 3 weeks post-stroke

ANS: A, B, D, E Clients most at risk for post-stroke depression are those with a previous history of depression, severe stroke (NIH Stroke Scale score of 38 is severe), and post-stroke physical or cognitive impairment. The client with mild forgetfulness and a slight limp would be a low priority for this referral.

A nurse is caring for a client who is on mechanical ventilation. What actions will promote comfort in this client? (Select all that apply.) a. Allow visitors at the client's bedside. b. Ensure the client can communicate if awake. c. Keep the television tuned to a favorite channel. d. Provide back and hand massages when turning. e. Turn the client every 2 hours or more.

ANS: A, B, D, E There are many basic care measures that can be employed for the client who is on a ventilator. Allowing visitation, providing a means of communication, massaging the client's skin, and routinely turning and repositioning the client are some of them. Keeping the TV on will interfere with sleep and rest.

A nurse is assessing a client with left-sided heart failure. For which clinical manifestations should the nurse assess? (Select all that apply.) a. Pulmonary crackles b. Confusion, restlessness c. Pulmonary hypertension d. Dependent edema e. Cough that worsens at night

ANS: A, B, E Left-sided heart failure occurs with a decrease in contractility of the heart or an increase in afterload. Most of the signs will be noted in the respiratory system. Right-sided heart failure occurs with problems from the pulmonary vasculature onward including pulmonary hypertension. Signs will be noted before the right atrium or ventricle including dependent edema.

A nurse plans care for a client who has hepatopulmonary syndrome. Which interventions should the nurse include in this client's plan of care? (Select all that apply.) a. Oxygen therapy b. Prone position c. Feet elevated on pillows d. Daily weights e. Physical therapy

ANS: A, C, D Care for a client who has hepatopulmonary syndrome should include oxygen therapy, the head of bed elevated at least 30 degrees or as high as the client wants to improve breathing, elevated feet to decrease dependent edema, and daily weights. There is no need to place the client in a prone position, on the client's stomach. Although physical therapy may be helpful to a client who has been hospitalized for several days, physical therapy is not an intervention specifically for hepatopulmonary syndrome.

A nurse cares for older clients who have traumatic brain injury. What should the nurse understand about this population? (Select all that apply.) a. Admission can overwhelm the coping mechanisms for older clients. b. Alcohol is typically involved in most traumatic brain injuries for this age group. c. These clients are more susceptible to systemic and wound infections. d. Other medical conditions can complicate treatment for these clients. e. Very few traumatic brain injuries occur in this age group.

ANS: A, C, D Older clients often tolerate stress poorly, which includes being admitted to a hospital that is unfamiliar and noisy. Because of decreased protective mechanisms, they are more susceptible to both local and systemic infections. Other medical conditions can complicate their treatment and recovery. Alcohol is typically not related to traumatic brain injury in this population; such injury is most often from falls and motor vehicle crashes. The 65- to 76-year-old age group has the second highest rate of brain injuries compared to other age groups.

A client is hospitalized in the oliguric phase of acute kidney injury (AKI) and is receiving tube feedings. The nurse is teaching the client's spouse about the kidney-specific formulation for the enteral solution compared to standard formulas. What components should be discussed in the teaching plan? (Select all that apply.) a. Lower sodium b. Higher calcium c. Lower potassium d. Higher phosphorus e. Higher calories

ANS: A, C, E Many clients with AKI are too ill to meet caloric goals and require tube feedings with kidney-specific formulas that are lower in sodium, potassium, and phosphorus, and higher in calories than are standard formulas.

A nurse is caring for a postoperative 70-kg client who had major blood loss during surgery. Which findings by the nurse should prompt immediate action to prevent acute kidney injury? (Select all that apply.) a. Urine output of 100 mL in 4 hours b. Urine output of 500 mL in 12 hours c. Large amount of sediment in the urine d. Amber, odorless urine e. Blood pressure of 90/60 mm Hg

ANS: A, C, E The low urine output, sediment, and blood pressure should be reported to the provider. Postoperatively, the nurse should measure intake and output, check the characteristics of the urine, and report sediment, hematuria, and urine output of less than 0.5 mL/kg/hour for 3 to 4 hours. A urine output of 100 mL is low, but a urine output of 500 mL in 12 hours should be within normal limits. Perfusion to the kidneys is compromised with low blood pressure. The amber odorless urine is normal.

A nursing student studying traumatic brain injuries (TBIs) should recognize which facts about these disorders? (Select all that apply.) a. A client with a moderate trauma may need hospitalization. b. A Glasgow Coma Scale score of 10 indicates a mild brain injury. c. Only open head injuries can cause a severe TBI. d. A client with a Glasgow Coma Scale score of 3 has severe TBI. e. The terms "mild TBI" and "concussion" have similar meanings.

ANS: A, D, E "Mild TBI" is a term used synonymously with the term "concussion." A moderate TBI has a Glasgow Coma Scale (GCS) score of 9 to 12, and these clients may need to be hospitalized. Both open and closed head injuries can cause a severe TBI, which is characterized by a GCS score of 3 to 8.

A nurse has applied to work at a hospital that has National Stroke Center designation. The nurse realizes the hospital adheres to eight Core Measures for ischemic stroke care. What do these Core Measures include? (Select all that apply.) a. Discharging the client on a statin medication b. Providing the client with comprehensive therapies c. Meeting goals for nutrition within 1 week d. Providing and charting stroke education e. Preventing venous thromboembolism

ANS: A, D, E Core Measures established by The Joint Commission include discharging stroke clients on statins, providing and recording stroke education, and taking measures to prevent venous thromboembolism. The client must be assessed for therapies but may go elsewhere for them. Nutrition goals are not part of the Core Measures.

A client received tissue plasminogen activator (t-PA) after a myocardial infarction and now is on an intravenous infusion of heparin. The client's spouse asks why the client needs this medication. What response by the nurse is best? a. "The t-PA didn't dissolve the entire coronary clot." b. "The heparin keeps that artery from getting blocked again." c. "Heparin keeps the blood as thin as possible for a longer time." d. "The heparin prevents a stroke from occurring as the t-PA wears off."

ANS: B After the original intracoronary clot has dissolved, large amounts of thrombin are released into the bloodstream, increasing the chance of the vessel reoccluding. The other statements are not accurate. Heparin is not a "blood thinner," although laypeople may refer to it as such.

A client is in the emergency department reporting a brief episode during which he was dizzy, unable to speak, and felt like his legs were very heavy. Currently the client's neurologic examination is normal. About what drug should the nurse plan to teach the client? a. Alteplase (Activase) b. Clopidogrel (Plavix) c. Heparin sodium d. Mannitol (Osmitrol)

ANS: B This client's manifestations are consistent with a transient ischemic attack, and the client would be prescribed aspirin or clopidogrel on discharge. Alteplase is used for ischemic stroke. Heparin and mannitol are not used for this condition.

A nurse teaches a client with heart failure about energy conservation. Which statement should the nurse include in this client's teaching? a. "Walk until you become short of breath, and then walk back home." b. "Gather everything you need for a chore before you begin." c. "Pull rather than push or carry items heavier than 5 pounds." d. "Take a walk after dinner every day to build up your strength."

ANS: B A client who has heart failure should be taught to conserve energy. Gathering all supplies needed for a chore at one time decreases the amount of energy needed. The client should not walk until becoming short of breath because he or she may not make it back home. Pushing a cart takes less energy than pulling or lifting. Although walking after dinner may help the client, the nurse should teach the client to complete activities when he or she has the most energy. This is usually in the morning.

A nurse cares for a client with hepatic portal-systemic encephalopathy (PSE). The client is thin and cachectic in appearance, and the family expresses distress that the client is receiving little dietary protein. How should the nurse respond? a. "A low-protein diet will help the liver rest and will restore liver function." b. "Less protein in the diet will help prevent confusion associated with liver failure." c. "Increasing dietary protein will help the client gain weight and muscle mass." d. "Low dietary protein is needed to prevent fluid from leaking into the abdomen."

ANS: B A low-protein diet is ordered when serum ammonia levels increase and/or the client shows signs of PSE. A low-protein diet helps reduce excessive breakdown of protein into ammonia by intestinal bacteria. Encephalopathy is caused by excess ammonia. A low-protein diet has no impact on restoring liver function. Increasing the client's dietary protein will cause complications of liver failure and should not be suggested. Increased intravascular protein will help prevent ascites, but clients with liver failure are not able to effectively synthesize dietary protein.

A nurse cares for a client who has cirrhosis of the liver. Which action should the nurse take to decrease the presence of ascites? a. Monitor intake and output. b. Provide a low-sodium diet. c. Increase oral fluid intake. d. Weigh the client daily.

ANS: B A low-sodium diet is one means of controlling abdominal fluid collection. Monitoring intake and output does not control fluid accumulation, nor does weighing the client. These interventions merely assess or monitor the situation. Increasing fluid intake would not be helpful.

A client has intra-arterial blood pressure monitoring after a myocardial infarction. The nurse notes the client's heart rate has increased from 88 to 110 beats/min, and the blood pressure dropped from 120/82 to 100/60 mm Hg. What action by the nurse is most appropriate? a. Allow the client to rest quietly. b. Assess the client for bleeding. c. Document the findings in the chart. d. Medicate the client for pain.

ANS: B A major complication related to intra-arterial blood pressure monitoring is hemorrhage from the insertion site. Since these vital signs are out of the normal range, are a change, and are consistent with blood loss, the nurse should assess the client for any bleeding associated with the arterial line. The nurse should document the findings after a full assessment. The client may or may not need pain medication and rest; the nurse first needs to rule out any emergent bleeding.

A nurse is caring for four clients on intravenous heparin therapy. Which laboratory value possibly indicates that a serious side effect has occurred? a. Hemoglobin: 14.2 g/dL b. Platelet count: 82,000/L 3 c. Red blood cell count: 4.8/mm 3 d. White blood cell count: 8.7/mm

ANS: B This platelet count is low and could indicate heparin-induced thrombocytopenia. The other values are normal for either gender.

A nurse obtains a client's health history at a community health clinic. Which statement alerts the nurse to provide health teaching to this client? a. "I drink two glasses of red wine each week." b. "I take a lot of Tylenol for my arthritis pain." c. "I have a cousin who died of liver cancer." d. "I got a hepatitis vaccine before traveling."

ANS: B Acetaminophen (Tylenol) can cause liver damage if taken in large amounts. Clients should be taught not to exceed 4000 mg/day of acetaminophen. The nurse should teach the client about this limitation and should explore other drug options with the client to manage his or her arthritis pain. Two glasses of wine each week, a cousin with liver cancer, and the hepatitis vaccine do not place the client at risk for a liver disorder, and therefore do not require any health teaching.

After administering newly prescribed captopril (Capoten) to a client with heart failure, the nurse implements interventions to decrease complications. Which priority intervention should the nurse implement for this client? a. Provide food to decrease nausea and aid in absorption. b. Instruct the client to ask for assistance when rising from bed. c. Collaborate with unlicensed assistive personnel to bathe the client. d. Monitor potassium levels and check for symptoms of hypokalemia.

ANS: B Administration of the first dose of angiotensin-converting enzyme (ACE) inhibitors is often associated with hypotension, usually termed first-dose effect. The nurse should instruct the client to seek assistance before arising from bed to prevent injury from postural hypotension. ACE inhibitors do not need to be taken with food. Collaboration with unlicensed assistive personnel to provide hygiene is not a priority. The client should be encouraged to complete activities of daily living as independently as possible. The nurse should monitor for hyperkalemia, not hypokalemia, especially if the client has renal insufficiency secondary to heart failure.

A client has an intraventricular catheter. What action by the nurse takes priority? a. Document intracranial pressure readings. b. Perform hand hygiene before client care. c. Measure intracranial pressure per hospital policy. d. Teach the client and family about the device.

ANS: B All of the actions are appropriate for this client. However, performing hand hygiene takes priority because it prevents infection, which is a possibly devastating complication.

A nurse assesses a client who has mitral valve regurgitation. For which cardiac dysrhythmia should the nurse assess? a. Preventricular contractions b. Atrial fibrillation c. Symptomatic bradycardia d. Sinus tachycardia

ANS: B Atrial fibrillation is a clinical manifestation of mitral valve regurgitation and stenosis. Preventricular contractions and bradycardia are not associated with valvular problems. These are usually identified in clients with electrolyte imbalances, myocardial infarction, and sinus node problems. Sinus tachycardia is a manifestation of aortic regurgitation due to a decrease in cardiac output.

A client is recovering from a kidney transplant. The client's urine output was 1500 mL over the last 12-hour period since transplantation. What is the priority assessment by the nurse? a. Checking skin turgor b. Taking blood pressure c. Assessing lung sounds d. Weighing the client

ANS: B By taking blood pressure, the nurse is assessing for hypotension that could compromise perfusion to the new kidney. The nurse then should notify the provider immediately. Skin turgor, lung sounds, and weight could give information about the fluid status of the client, but they are not the priority assessment.

After teaching a client who is being discharged home after mitral valve replacement surgery, the nurse assesses the client's understanding. Which client statement indicates a need for additional teaching? a. "I'll be able to carry heavy loads after 6 months of rest." b. "I will have my teeth cleaned by my dentist in 2 weeks." c. "I must avoid eating foods high in vitamin K, like spinach." d. "I must use an electric razor instead of a straight razor to shave."

ANS: B Clients who have defective or repaired valves are at high risk for endocarditis. The client who has had valve surgery should avoid dental procedures for 6 months because of the risk for endocarditis. When undergoing a mitral valve replacement surgery, the client needs to be placed on anticoagulant therapy to prevent vegetation forming on the new valve. Clients on anticoagulant therapy should be instructed on bleeding precautions, including using an electric razor. If the client is prescribed warfarin, the client should avoid foods high in vitamin K. Clients recovering from open heart valve replacements should not carry anything heavy for 6 months while the chest incision and muscle heal.

A nurse assesses a client who has a history of heart failure. Which question should the nurse ask to assess the extent of the client's heart failure? a. "Do you have trouble breathing or chest pain?" b. "Are you able to walk upstairs without fatigue?" c. "Do you awake with breathlessness during the night?" d. "Do you have new-onset heaviness in your legs?"

ANS: B Clients with a history of heart failure generally have negative findings, such as shortness of breath. The nurse needs to determine whether the client's activity is the same or worse, or whether the client identifies a decrease in activity level. Trouble breathing, chest pain, breathlessness at night, and peripheral edema are symptoms of heart failure, but do not provide data that can determine the extent of the client's heart failure.

An emergency department nurse assesses a client with kidney trauma and notes that the client's abdomen is tender and distended and blood is visible at the urinary meatus. Which prescription should the nurse consult the provider about before implementation? a. Assessing vital signs every 15 minutes b. Inserting an indwelling urinary catheter c. Administering intravenous fluids at 125 mL/hr d. Typing and crossmatching for blood products

ANS: B Clients with blood at the urinary meatus should not have a urinary catheter inserted via the urethra before additional diagnostic studies are done. The urethra could be torn. The nurse should question the provider about the need for a catheter; if one is needed, the provider can insert a suprapubic catheter. The nurse should monitor the client's vital signs closely, send blood for type and crossmatch in case the client needs blood products, and administer intravenous fluids.

A client with a traumatic brain injury is agitated and fighting the ventilator. What drug should the nurse prepare to administer? a. Carbamazepine (Tegretol) b. Dexmedetomidine (Precedex) c. Diazepam (Valium) d. Mannitol (Osmitrol)

ANS: B Dexmedetomidine is often used to manage agitation in the client with traumatic brain injury. Carbamazepine is an antiseizure drug. Diazepam is a benzodiazepine. Mannitol is an osmotic diuretic.

The nurse is teaching the main principles of hemodialysis to a client with chronic kidney disease. Which statement by the client indicates a need for further teaching by the nurse? a. "My sodium level changes by movement from the blood into the dialysate." b. "Dialysis works by movement of wastes from lower to higher concentration." c. "Extra fluid can be pulled from the blood by osmosis." d. "The dialysate is similar to blood but without any toxins."

ANS: B Dialysis works using the passive transfer of toxins by diffusion. Diffusion is the movement of molecules from an area of higher concentration to an area of lower concentration. The other statements show a correct understanding about hemodialysis.

A nurse assesses a client with mitral valve stenosis. What clinical manifestation should alert the nurse to the possibility that the client's stenosis has progressed? a. Oxygen saturation of 92% b. Dyspnea on exertion c. Muted systolic murmur d. Upper extremity weakness

ANS: B Dyspnea on exertion develops as the mitral valvular orifice narrows and pressure in the lungs increases. The other manifestations do not relate to the progression of mitral valve stenosis.

A client is on intravenous heparin to treat a pulmonary embolism. The client's most recent partial thromboplastin time (PTT) was 25 seconds. What order should the nurse anticipate? a. Decrease the heparin rate. b. Increase the heparin rate. c. No change to the heparin rate. d. Stop heparin; start warfarin (Coumadin).

ANS: B For clients on heparin, a PTT of 1.5 to 2.5 times the normal value is needed to demonstrate the heparin is working. A normal PTT is 25 to 35 seconds, so this client's PTT value is too low. The heparin rate needs to be increased. Warfarin is not indicated in this situation.

A client is being prepared for a mechanical embolectomy. What action by the nurse takes priority? a. Assess for contraindications to fibrinolytics. b. Ensure that informed consent is on the chart. c. Perform a full neurologic assessment. d. Review the client's medication lists.

ANS: B For this invasive procedure, the client needs to give informed consent. The nurse ensures that this is on the chart prior to the procedure beginning. Fibrinolytics are not used. A neurologic assessment and medication review are important, but the consent is the priority.

A nurse is preparing to admit a client on mechanical ventilation from the emergency department. What action by the nurse takes priority? a. Assessing that the ventilator settings are correct b. Ensuring there is a bag-valve-mask in the room c. Obtaining personal protective equipment d. Planning to suction the client upon arrival to the room

ANS: B Having a bag-valve-mask device is critical in case the client needs manual breathing. The respiratory therapist is usually primarily responsible for setting up the ventilator, although the nurse should know and check the settings. Personal protective equipment is important, but ensuring client safety takes priority. The client may or may not need suctioning on arrival.

A nurse is in charge of the coronary intensive care unit. Which client should the nurse see first? a. Client on a nitroglycerin infusion at 5 mcg/min, not titrated in the last 4 hours b. Client who is 1 day post coronary artery bypass graft, blood pressure 180/100 mm Hg c. Client who is 1 day post percutaneous coronary intervention, going home this morning d. Client who is 2 days post coronary artery bypass graft, became dizzy this a.m. while walking

ANS: B Hypertension after coronary artery bypass graft surgery can be dangerous because it puts too much pressure on the suture lines and can cause bleeding. The charge nurse should see this client first. The client who became dizzy earlier should be seen next. The client on the nitroglycerin drip is stable. The client going home can wait until the other clients are cared for.

A client has hemodynamic monitoring after a myocardial infarction. What safety precaution does the nurse implement for this client? a. Document pulmonary artery wedge pressure (PAWP) readings and assess their trends. b. Ensure the balloon does not remain wedged. c. Keep the client on strict NPO status. d. Maintain the client in a semi-Fowler's position.

ANS: B If the balloon remains inflated, it can cause pulmonary infarction or rupture. The nurse should ensure the balloon remains deflated between PAWP readings. Documenting PAWP readings and assessing trends is an important nursing action related to hemodynamic monitoring, but is not specifically related to safety. The client does not have to be NPO while undergoing hemodynamic monitoring. Positioning may or may not affect readings.

The nurse is caring for a client with a chest tube after a coronary artery bypass graft. The drainage slows significantly. What action by the nurse is most important? a. Increase the setting on the suction. b. Notify the provider immediately. c. Re-position the chest tube. d. Take the tubing apart to assess for clots.

ANS: B If the drainage in the chest tube decreases significantly and dramatically, the tube may be blocked by a clot. This could lead to cardiac tamponade. The nurse should notify the provider immediately. The nurse should not independently increase the suction, re-position the chest tube, or take the tubing apart.

A nurse is caring for four clients who might be brain dead. Which client would best meet the criteria to allow assessment of brain death? a. Client with a core temperature of 95° F (35° C) for 2 days b. Client in a coma for 2 weeks from a motor vehicle crash c. Client who is found unresponsive in a remote area of a field by a hunter d. Client with a systolic blood pressure of 92 mm Hg since admission

ANS: B In order to determine brain death, clients must meet four criteria: 1) coma from a known cause, 2) normal or near-normal core temperature, 3) normal systolic blood pressure, and 4) at least one neurologic examination. The client who was in the car crash meets two of these criteria. The clients with the lower temperature and lower blood pressure have only one of these criteria. There is no data to support assessment of brain death in the client found by the hunter.

The nurse is caring for four clients with chronic kidney disease. Which client should the nurse assess first upon initial rounding? a. Woman with a blood pressure of 158/90 mm Hg b. Client with Kussmaul respirations c. Man with skin itching from head to toe d. Client with halitosis and stomatitis

ANS: B Kussmaul respirations indicate a worsening of chronic kidney disease (CKD). The client is increasing the rate and depth of breathing to excrete carbon dioxide through the lungs. Hypertension is common in most clients with CKD, and skin itching increases with calcium-phosphate imbalances, another common finding in CKD. Uremia from CKD causes ammonia to be formed, resulting in the common findings of halitosis and stomatitis.

A client is hospitalized with a second episode of pulmonary embolism (PE). Recent genetic testing reveals the client has an alteration in the gene CYP2C19. What action by the nurse is best? a. Instruct the client to eliminate all vitamin K from the diet. b. Prepare preoperative teaching for an inferior vena cava (IVC) filter. c. Refer the client to a chronic illness support group. d. Teach the client to use a soft-bristled toothbrush.

ANS: B Often clients are discharged from the hospital on warfarin (Coumadin) after a PE. However, clients with a variation in the CYP2C19 gene do not metabolize warfarin well and have higher blood levels and more side effects. This client is a poor candidate for warfarin therapy, and the prescriber will most likely order an IVC filter device to be implanted. The nurse should prepare to do preoperative teaching on this procedure. It would be impossible to eliminate all vitamin K from the diet. A chronic illness support group may be needed, but this is not the best intervention as it is not as specific to the client as the IVC filter. A soft-bristled toothbrush is a safety measure for clients on anticoagulation therapy.

A nurse assesses a client with polycystic kidney disease (PKD). Which assessment finding should alert the nurse to immediately contact the health care provider? a. Flank pain b. Periorbital edema c. Bloody and cloudy urine d. Enlarged abdomen

ANS: B Periorbital edema would not be a finding related to PKD and should be investigated further. Flank pain and a distended or enlarged abdomen occur in PKD because the kidneys enlarge and displace other organs. Urine can be bloody or cloudy as a result of cyst rupture or infection.

The nurse is assessing a client with a diagnosis of pre-renal acute kidney injury (AKI). Which condition would the nurse expect to find in the client's recent history? a. Pyelonephritis b. Myocardial infarction c. Bladder cancer d. Kidney stones

ANS: B Pre-renal causes of AKI are related to a decrease in perfusion, such as with a myocardial infarction. Pyelonephritis is an intrinsic or intrarenal cause of AKI related to kidney damage. Bladder cancer and kidney stones are post-renal causes of AKI related to urine flow obstruction.

A student nurse is preparing morning medications for a client who had a stroke. The student plans to hold the docusate sodium (Colace) because the client had a large stool earlier. What action by the supervising nurse is best? a. Have the student ask the client if it is desired or not. b. Inform the student that the docusate should be given. c. Tell the student to document the rationale. d. Tell the student to give it unless the client refuses.

ANS: B Stool softeners should be given to clients with neurologic disorders in order to prevent an elevation in intracranial pressure that accompanies the Valsalva maneuver when constipated. The supervising nurse should instruct the student to administer the docusate. The other options are not appropriate. The medication could be held for diarrhea.

A nurse cares for a client recovering from prosthetic valve replacement surgery. The client asks, "Why will I need to take anticoagulants for the rest of my life?" How should the nurse respond? a. "The prosthetic valve places you at greater risk for a heart attack." b. "Blood clots form more easily in artificial replacement valves." c. "The vein taken from your leg reduces circulation in the leg." d. "The surgery left a lot of small clots in your heart and lungs."

ANS: B Synthetic valve prostheses and scar tissue provide surfaces on which platelets can aggregate easily and initiate the formation of blood clots. The other responses are inaccurate.

A client's mean arterial pressure is 60 mm Hg and intracranial pressure is 20 mm Hg. Based on the client's cerebral perfusion pressure, what should the nurse anticipate for this client? a. Impending brain herniation b. Poor prognosis and cognitive function c. Probable complete recovery d. Unable to tell from this information

ANS: B The cerebral perfusion pressure (CPP) is the intracranial pressure subtracted from the mean arterial pressure: in this case, 60 - 20 = 40. For optimal outcomes, CPP should be at least 70 mm Hg. This client has very low CPP, which will probably lead to a poorer prognosis with significant cognitive dysfunction should the client survive. This data does not indicate impending brain herniation or complete recovery.

After teaching a client with hypertension secondary to renal disease, the nurse assesses the client's understanding. Which statement made by the client indicates a need for additional teaching? a. "I can prevent more damage to my kidneys by managing my blood pressure." b. "If I have increased urination at night, I need to drink less fluid during the day." c. "I need to see the registered dietitian to discuss limiting my protein intake." d. "It is important that I take my antihypertensive medications as directed."

ANS: B The client should not restrict fluids during the day due to increased urination at night. Clients with renal disease may be prescribed fluid restrictions. These clients should be assessed thoroughly for potential dehydration. Increased nocturnal voiding can be decreased by consuming fluids earlier in the day. Blood pressure control is needed to slow the progression of renal dysfunction. When dietary protein is restricted, refer the client to the registered dietitian as needed.

A nurse assesses a client with pericarditis. Which assessment finding should the nurse expect to find? a. Heart rate that speeds up and slows down b. Friction rub at the left lower sternal border c. Presence of a regular gallop rhythm d. Coarse crackles in bilateral lung bases

ANS: B The client with pericarditis may present with a pericardial friction rub at the left lower sternal border. This sound is the result of friction from inflamed pericardial layers when they rub together. The other assessments are not related.

When the nursing supervisor is evaluating the performance of a new RN, which action indicates that the new RN is safe in providing care to a patient who is receiving mechanical ventilation with 10 cm of peak end-expiratory pressure (PEEP)? a. The RN plans to suction the patient every 2 hours. b. The RN uses a closed-suction technique to suction the patient. c. The RN tapes connection between the ventilator tubing and the ET. d. The RN changes the ventilator circuit tubing routinely every 24 hours.

ANS: B The closed-suction technique is suggested when patients require high levels of PEEP to prevent the loss of PEEP that occurs when disconnecting the patient from the ventilator. Suctioning should not be scheduled routinely, but it should be done only when patient assessment data indicate the need for suctioning. Taping connections between the ET and the ventilator tubing would restrict the ability of the tubing to swivel in response to patient repositioning. Ventilator tubing changes increase the risk for ventilator-associated pneumonia (VAP) and are not indicated routinely. DIF: Cognitive Level: Application REF: 1703-1704 OBJ: Special Questions: Delegation TOP: Nursing Process: Implementation MSC: NCLEX: Safe and Effective Care Environment

A patient is admitted to the emergency department with an open stab wound to the left chest. What is the first action that the nurse should take? a. Position the patient so that the left chest is dependent. b. Tape a nonporous dressing on three sides over the chest wound. c. Cover the sucking chest wound firmly with an occlusive dressing. d. Keep the head of the patient's bed at no more than 30 degrees elevation.

ANS: B The dressing taped on three sides will allow air to escape when intrapleural pressure increases during expiration, but it will prevent air from moving into the pleural space during inspiration. Placing the patient on the left side or covering the chest wound with an occlusive dressing will allow trapped air in the pleural space and cause tension pneumothorax. The head of the bed should be elevated to 30 to 45 degrees to facilitate breathing.

A patient who is receiving mechanical ventilation is anxious and is "fighting" the ventilator. Which action should the nurse take first? a. Ventilate the patient with a manual resuscitation bag. b. Verbally coach the patient to breathe with the ventilator. c. Sedate the patient with the ordered PRN lorazepam (Ativan). d. Increase the rate for the ordered propofol (Diprivan) infusion.

ANS: B The initial response by the nurse should be to try to decrease the patient's anxiety by coaching the patient about how to coordinate respirations with the ventilator. The other actions also may be helpful if the verbal coaching is ineffective in reducing the patient's anxiety. DIF: Cognitive Level: Application REF: 1704-1705 OBJ: Special Questions: Prioritization TOP: Nursing Process: Implementation MSC: NCLEX: Physiological Integrity

The nurse monitors a patient after chest tube placement for a hemopneumothorax. The nurse is most concerned if which assessment finding is observed? a. A large air leak in the water-seal chamber b. 400 mL of blood in the collection chamber c. Complaint of pain with each deep inspiration d. Subcutaneous emphysema at the insertion site

ANS: B The large amount of blood may indicate that the patient is in danger of developing hypovolemic shock. An air leak would be expected immediately after chest tube placement for a pneumothorax. Initially, brisk bubbling of air occurs in this chamber when a pneumothorax is evacuated. The pain should be treated but is not as urgent a concern as the possibility of continued hemorrhage. Subcutaneous emphysema should be monitored but is not unusual in a patient with pneumothorax. A small amount of subcutaneous air is harmless and will be reabsorbed.

An emergency room nurse assesses a client after a motor vehicle crash. The nurse notices a "steering wheel mark" across the client's chest. Which action should the nurse take? a. Ask the client where in the car he or she was sitting during the crash. b. Assess the client by gently palpating the abdomen for tenderness. c. Notify the laboratory to draw blood for blood type and crossmatch. d. Place the client on the stretcher in reverse Trendelenburg position.

ANS: B The liver is often injured by a steering wheel in a motor vehicle crash. Because the client's chest was marked by the steering wheel, the nurse should perform an abdominal assessment. Assessing the client's position in the crash is not needed because of the steering wheel imprint. The client may or may not need a blood transfusion. The client does not need to be in reverse Trendelenburg position.

Four hours after mechanical ventilation is initiated for a patient with chronic obstructive pulmonary disease (COPD), the patient's arterial blood gas (ABG) results include a pH of 7.50, PaO2 of 80 mm Hg, PaCO2 of 29 mm Hg, and HCO3- of 23 mEq/L (23 mmol/L). The nurse will anticipate the need to a. increase the FIO2. b. decrease the respiratory rate. c. increase the tidal volume (VT). d. leave the ventilator at the current settings.

ANS: B The patient's PaCO2 and pH indicate respiratory alkalosis caused by too high a respiratory rate. The PaO2 is appropriate for a patient with COPD, increasing the tidal volume would further lower the PaCO2, and the PaCO2 and pH indicate a need to make the ventilator changes. DIF: Cognitive Level: Analysis REF: 1710-1711 TOP: Nursing Process: Planning MSC: NCLEX: Physiological Integrity

A nurse is caring for four clients. Which client should the nurse assess first? a. Client with an acute myocardial infarction, pulse 102 beats/min b. Client who is 1 hour post angioplasty, has tongue swelling and anxiety c. Client who is post coronary artery bypass, chest tube drained 100 mL/hr d. Client who is post coronary artery bypass, potassium 4.2 mEq/L

ANS: B The post-angioplasty client with tongue swelling and anxiety is exhibiting manifestations of an allergic reaction that could progress to anaphylaxis. The nurse should assess this client first. The client with a heart rate of 102 beats/min may have increased oxygen demands but is just over the normal limit for heart rate. The two post coronary artery bypass clients are stable.

A nurse answers a call light and finds a client anxious, short of breath, reporting chest pain, and having a blood pressure of 88/52 mm Hg on the cardiac monitor. What action by the nurse takes priority? a. Assess the client's lung sounds. b. Notify the Rapid Response Team. c. Provide reassurance to the client. d. Take a full set of vital signs.

ANS: B This client has manifestations of a pulmonary embolism, and the most critical action is to notify the Rapid Response Team for speedy diagnosis and treatment. The other actions are appropriate also but are not the priority.

After a craniotomy, the nurse assesses the client and finds dry, sticky mucous membranes and restlessness. The client has IV fluids running at 75 mL/hr. What action by the nurse is best? a. Assess the client's magnesium level. b. Assess the client's sodium level. c. Increase the rate of the IV infusion. d. Provide oral care every hour.

ANS: B This client has manifestations of hypernatremia, which is a possible complication after craniotomy. The nurse should assess the client's serum sodium level. Magnesium level is not related. The nurse does not independently increase the rate of the IV infusion. Providing oral care is also a good option but does not take priority over assessing laboratory results.

A client is admitted with acute kidney injury (AKI) and a urine output of 2000 mL/day. What is the major concern of the nurse regarding this client's care? a. Edema and pain b. Electrolyte and fluid imbalance c. Cardiac and respiratory status d. Mental health status

ANS: B This client may have an inflammatory cause of AKI with proteins entering the glomerulus and holding the fluid in the filtrate, causing polyuria. Electrolyte loss and fluid balance is essential. Edema and pain are not usually a problem with fluid loss. There could be changes in the client's cardiac, respiratory, and mental health status if the electrolyte imbalance is not treated.

A client is in the hospital after suffering a myocardial infarction and has bathroom privileges. The nurse assists the client to the bathroom and notes the client's O2 saturation to be 95%, pulse 88 beats/min, and respiratory rate 16 breaths/min after returning to bed. What action by the nurse is best? a. Administer oxygen at 2 L/min. b. Allow continued bathroom privileges. c. Obtain a bedside commode. d. Suggest the client use a bedpan.

ANS: B This client's physiologic parameters did not exceed normal during and after activity, so it is safe for the client to continue using the bathroom. There is no indication that the client needs oxygen, a commode, or a bedpan.

A nurse is dismissing a client from the emergency department who has a mild traumatic brain injury. What information obtained from the client represents a possible barrier to self-management? (Select all that apply.) a. Does not want to purchase a thermometer b. Is allergic to acetaminophen (Tylenol) c. Laughing, says "Strenuous? What's that?" d. Lives alone and is new in town with no friends e. Plans to have a beer and go to bed once home

ANS: B, D, E Clients should take acetaminophen for headache. An allergy to this drug may mean the client takes aspirin or ibuprofen (Motrin), which should be avoided. The client needs neurologic checks every 1 to 2 hours, and this client does not seem to have anyone available who can do that. Alcohol needs to be avoided for at least 24 hours. A thermometer is not needed. The client laughing at strenuous activity probably does not engage in any kind of strenuous activity, but the nurse should confirm this.

A nurse is caring for a client after a stroke. What actions may the nurse delegate to the unlicensed assistive personnel (UAP)? (Select all that apply.) a. Assess neurologic status with the Glasgow Coma Scale. b. Check and document oxygen saturation every 1 to 2 hours. c. Cluster client care to allow periods of uninterrupted rest. d. Elevate the head of the bed to 45 degrees to prevent aspiration. e. Position the client supine with the head in a neutral midline position.

ANS: B, E The UAP can take and document vital signs, including oxygen saturation, and keep the client's head in a neutral, midline position with correct direction from the nurse. The nurse assesses the Glasgow Coma Scale score. The nursing staff should not cluster care because this can cause an increase in the intracranial pressure. The head of the bed should be minimally elevated, up to 30 degrees.

A client is receiving an infusion of tissue plasminogen activator (t-PA). The nurse assesses the client to be disoriented to person, place, and time. What action by the nurse is best? a. Assess the client's pupillary responses. b. Request a neurologic consultation. c. Stop the infusion and call the provider. d. Take and document a full set of vital signs.

ANS: C A change in neurologic status in a client receiving t-PA could indicate intracranial hemorrhage. The nurse should stop the infusion and notify the provider immediately. A full assessment, including pupillary responses and vital signs, occurs next. The nurse may or may not need to call a neurologist.

A client has a pulmonary embolism and is started on oxygen. The student nurse asks why the client's oxygen saturation has not significantly improved. What response by the nurse is best? a. "Breathing so rapidly interferes with oxygenation." b. "Maybe the client has respiratory distress syndrome." c. "The blood clot interferes with perfusion in the lungs." d. "The client needs immediate intubation and mechanical ventilation."

ANS: C A large blood clot in the lungs will significantly impair gas exchange and oxygenation. Unless the clot is dissolved, this process will continue unabated. Hyperventilation can interfere with oxygenation by shallow breathing, but there is no evidence that the client is hyperventilating, and this is also not the most precise physiologic answer. Respiratory distress syndrome can occur, but this is not as likely. The client may need to be mechanically ventilated, but without concrete data on FiO2 and SaO2, the nurse cannot make that judgment.

The nurse notes that a patient has incisional pain, a poor cough effort, and scattered rhonchi after a thoracotomy. Which action should the nurse take first? a. Assist the patient to sit upright in a chair. b. Splint the patient's chest during coughing. c. Medicate the patient with prescribed morphine. d. Observe the patient use the incentive spirometer.

ANS: C A major reason for atelectasis and poor airway clearance in patients after chest surgery is incisional pain (which increases with deep breathing and coughing). The first action by the nurse should be to medicate the patient to minimize incisional pain. The other actions are all appropriate ways to improve airway clearance but should be done after the morphine is given.

A client in the cardiac stepdown unit reports severe, crushing chest pain accompanied by nausea and vomiting. What action by the nurse takes priority? a. Administer an aspirin. b. Call for an electrocardiogram (ECG). c. Maintain airway patency. d. Notify the provider.

ANS: C Airway always is the priority. The other actions are important in this situation as well, but the nurse should stay with the client and ensure the airway remains patent (especially if vomiting occurs) while another person calls the provider (or Rapid Response Team) and facilitates getting an ECG done. Aspirin will probably be administered, depending on the provider's prescription and the client's current medications.

After teaching a client who is recovering from a heart transplant to change positions slowly, the client asks, "Why is this important?" How should the nurse respond? a. "Rapid position changes can create shear and friction forces, which can tear out your internal vascular sutures." b. "Your new vascular connections are more sensitive to position changes, leading to increased intravascular pressure and dizziness." c. "Your new heart is not connected to the nervous system and is unable to respond to decreases in blood pressure caused by position changes." d. "While your heart is recovering, blood flow is diverted away from the brain, increasing the risk for stroke when you stand up."

ANS: C Because the new heart is denervated, the baroreceptor and other mechanisms that compensate for blood pressure drops caused by position changes do not function. This allows orthostatic hypotension to persist in the postoperative period. The other options are false statements and do not correctly address the client's question.

The nurse notes thick, white respiratory secretions for a patient who is receiving mechanical ventilation. Which intervention will be most effective in resolving this problem? a. Suction the patient every hour. b. Reposition the patient every 2 hours. c. Add additional water to the patient's enteral feedings. d. Instill 5 mL of sterile saline into the endotracheal tube (ET) before suctioning.

ANS: C Because the patient's secretions are thick, better hydration is indicated. Suctioning every hour without any specific evidence for the need will increase the incidence of mucosal trauma and would not address the etiology of the ineffective airway clearance. Instillation of saline does not liquefy secretions and may decrease the SpO2. Repositioning the patient is appropriate but will not decrease the thickness of secretions. DIF: Cognitive Level: Application REF: 1703-1704 TOP: Nursing Process: Implementation MSC: NCLEX: Physiological Integrity

A nurse assesses a client in an outpatient clinic. Which statement alerts the nurse to the possibility of left-sided heart failure? a. "I have been drinking more water than usual." b. "I am awakened by the need to urinate at night." c. "I must stop halfway up the stairs to catch my breath." d. "I have experienced blurred vision on several occasions."

ANS: C Clients with left-sided heart failure report weakness or fatigue while performing normal activities of daily living, as well as difficulty breathing, or "catching their breath." This occurs as fluid moves into the alveoli. Nocturia is often seen with right-sided heart failure. Thirst and blurred vision are not related to heart failure.

A patient who has a right-sided chest tube following a thoracotomy has continuous bubbling in the suction-control chamber of the collection device. Which action by the nurse is most appropriate? a. Document the presence of a large air leak. b. Notify the surgeon of a possible pneumothorax. c. Take no further action with the collection device. d. Adjust the dial on the wall regulator to decrease suction.

ANS: C Continuous bubbling is expected in the suction-control chamber and indicates that the suction-control chamber is connected to suction. An air leak would be detected in the water-seal chamber. There is no evidence of pneumothorax. Increasing or decreasing the vacuum source will not adjust the suction pressure. The amount of suction applied is regulated by the amount of water in this chamber and not by the amount of suction applied to the system.

The nurse is taking the vital signs of a client after hemodialysis. Blood pressure is 110/58 mm Hg, pulse 66 beats/min, and temperature is 99.8° F (37.6° C). What is the most appropriate action by the nurse? a. Administer fluid to increase blood pressure. b. Check the white blood cell count. c. Monitor the client's temperature. d. Connect the client to an electrocardiographic (ECG) monitor.

ANS: C During hemodialysis, the dialysate is warmed to increase diffusion and prevent hypothermia. The client's temperature could reflect the temperature of the dialysate. There is no indication to check the white blood cell count or connect the client to an ECG monitor. The other vital signs are within normal limits.

A nurse is assisting the health care provider who is intubating a client. The provider has been attempting to intubate for 40 seconds. What action by the nurse takes priority? a. Ensure the client has adequate sedation. b. Find another provider to intubate. c. Interrupt the procedure to give oxygen. d. Monitor the client's oxygen saturation.

ANS: C Each intubation attempt should not exceed 30 seconds (15 is preferable) as it causes hypoxia. The nurse should interrupt the intubation attempt and give the client oxygen. The nurse should also have adequate sedation during the procedure and monitor the client's oxygen saturation, but these do not take priority. Finding another provider is not appropriate at this time.

A client is admitted with a pulmonary embolism (PE). The client is young, healthy, and active and has no known risk factors for PE. What action by the nurse is most appropriate? a. Encourage the client to walk 5 minutes each hour. b. Refer the client to smoking cessation classes. c. Teach the client about factor V Leiden testing. d. Tell the client that sometimes no cause for disease is found.

ANS: C Factor V Leiden is an inherited thrombophilia that can lead to abnormal clotting events, including PE. A client with no known risk factors for this disorder should be referred for testing. Encouraging the client to walk is healthy, but is not related to the development of a PE in this case, nor is smoking. Although there are cases of disease where no cause is ever found, this assumption is premature.

A client experiences impaired swallowing after a stroke and has worked with speech-language pathology on eating. What nursing assessment best indicates that a priority goal for this problem has been met? a. Chooses preferred items from the menu b. Eats 75% to 100% of all meals and snacks c. Has clear lung sounds on auscultation d. Gains 2 pounds after 1 week

ANS: C Impaired swallowing can lead to aspiration, so the priority goal for this problem is no aspiration. Clear lung sounds is the best indicator that aspiration has not occurred. Choosing menu items is not related to this problem. Eating meals does not indicate the client is not still aspirating. A weight gain indicates improved nutrition but still does not show a lack of aspiration.

The nurse is teaching a client how to increase the flow of dialysate into the peritoneal cavity during dialysis. Which statement by the client demonstrates a correct understanding of the teaching? a. "I should leave the drainage bag above the level of my abdomen." b. "I could flush the tubing with normal saline if the flow stops." c. "I should take a stool softener every morning to avoid constipation." d. "My diet should have low fiber in it to prevent any irritation."

ANS: C Inflow and outflow problems of the dialysate are best controlled by preventing constipation. A daily stool softener is the best option for the client. The drainage bag should be below the level of the abdomen. Flushing the tubing will not help with the flow. A diet high in fiber will also help with a constipation problem.

The family members of a patient who has just been admitted to the intensive care unit (ICU) with multiple traumatic injuries have just arrived in the ICU waiting room. Which action should the nurse take first? a. Immediately take the family members to the patient's room. b. Discuss ICU visitation policies and encourage family visits. c. Describe the patient's injuries and the care that is being provided. d. Invite the family to participate in a multidisciplinary care conference.

ANS: C Lack of information is a major source of anxiety for family members and should be addressed first. Family members should be prepared for the patient's appearance and the ICU environment before visiting the patient for the first time. ICU visiting should be individualized to each patient and family rather than being dictated by rigid visitation policies. Inviting the family to participate in a multidisciplinary conference is appropriate but should not be the initial action by the nurse. DIF: Cognitive Level: Application REF: 1686-1687 OBJ: Special Questions: Prioritization TOP: Nursing Process: Implementation MSC: NCLEX: Psychosocial Integrity

A client had an acute myocardial infarction. What assessment finding indicates to the nurse that a significant complication has occurred? a. Blood pressure that is 20 mm Hg below baseline b. Oxygen saturation of 94% on room air c. Poor peripheral pulses and cool skin d. Urine output of 1.2 mL/kg/hr for 4 hours

ANS: C Poor peripheral pulses and cool skin may be signs of impending cardiogenic shock and should be reported immediately. A blood pressure drop of 20 mm Hg is not worrisome. An oxygen saturation of 94% is just slightly below normal. A urine output of 1.2 mL/kg/hr for 4 hours is normal.

When the nurse is weaning a patient who has chronic obstructive pulmonary disease (COPD) from mechanical ventilation, which patient assessment indicates that the weaning protocol should be discontinued? a. The patient heart rate is 98 beats/min. b. The patient's oxygen saturation is 93%. c. The patient respiratory rate is 32 breaths/min. d. The patient's spontaneous tidal volume is 500 mL.

ANS: C Tachypnea is a sign that the patient's work of breathing is too high to allow weaning to proceed. The patient's heart rate is within normal limits, although the nurse should continue to monitor it. An oxygen saturation of 93% is acceptable for a patient with COPD. A spontaneous tidal volume of 500 mL is within the acceptable range. DIF: Cognitive Level: Application REF: 1713 TOP: Nursing Process: Evaluation MSC: NCLEX: Physiological Integrity

A client presents to the emergency department with an acute myocardial infarction (MI) at 1500 (3:00 PM). The facility has 24-hour catheterization laboratory abilities. To meet The Joint Commission's Core Measures set, by what time should the client have a percutaneous coronary intervention performed? a. 1530 (3:30 PM) b. 1600 (4:00 PM) c. 1630 (4:30 PM) d. 1700 (5:00 PM)

ANS: C The Joint Commission's Core Measures set for MI includes percutaneous coronary intervention within 90 minutes of diagnosis of myocardial infarction. Therefore, the client should have a percutaneous coronary intervention performed no later than 1630 (4:30 PM).

A nurse is caring for four clients in the neurologic/neurosurgical intensive care unit. Which client should the nurse assess first? a. Client who has been diagnosed with meningitis with a fever of 101° F (38.3° C) b. Client who had a transient ischemic attack and is waiting for teaching on clopidogrel (Plavix) c. Client receiving tissue plasminogen activator (t-PA) who has a change in respiratory pattern and rate d. Client who is waiting for subarachnoid bolt insertion with the consent form already signed

ANS: C The client receiving t-PA has a change in neurologic status while receiving this fibrinolytic therapy. The nurse assesses this client first as he or she may have an intracerebral bleed. The client with meningitis has expected manifestations. The client waiting for discharge teaching is a lower priority. The client waiting for surgery can be assessed quickly after the nurse sees the client who is receiving t-PA, or the nurse could delegate checking on this client to another nurse.

When the charge nurse is evaluating the care that a new RN staff member provides to a patient receiving mechanical ventilation, which action by the new RN indicates the need for more education? a. The RN turns the FIO2 up to 100% before suctioning. b. The RN secures a bite block in place using adhesive tape. c. The RN positions the patient with the head of bed at 10 degrees. d. The RN asks for assistance to turn the patient to the prone position.

ANS: C The head of the patient's bed should be positioned at 30 to 45 degrees to prevent ventilator-acquired pneumonia. The other actions by the new RN are appropriate. DIF: Cognitive Level: Application REF: 1711 OBJ: Special Questions: Delegation TOP: Nursing Process: Evaluation MSC: NCLEX: Safe and Effective Care Environment

While assessing a patient with a central venous catheter, the nurse notes the catheter insertion site is red and tender and the patient's temperature is 101.8° F. The nurse will plan to a. administer analgesics and antibiotics. b. check the site frequently for any swelling. c. discontinue the catheter and culture the tip. d. change the flush system and monitor the site.

ANS: C The information indicates that the patient has a local and systemic infection caused by the catheter and the catheter should be discontinued. Changing the flush system, administration of analgesics, and continued monitoring will not help prevent or treat the infection. Administration of antibiotics is appropriate, but the line should still be discontinued to avoid further complications such as endocarditis. DIF: Cognitive Level: Application REF: 1696 TOP: Nursing Process: Planning MSC: NCLEX: Physiological Integrity

The charge nurse is orienting a float nurse to an assigned client with an arteriovenous (AV) fistula for hemodialysis in her left arm. Which action by the float nurse would be considered unsafe? a. Palpating the access site for a bruit or thrill b. Using the right arm for a blood pressure reading c. Administering intravenous fluids through the AV fistula d. Checking distal pulses in the left arm

ANS: C The nurse should not use the arm with the AV fistula for intravenous infusion, blood pressure readings, or venipuncture. Compression and infection can result in the loss of the AV fistula. The AV fistula should be monitored by auscultating or palpating the access site. Checking the distal pulse would be an appropriate assessment.

A client has a traumatic brain injury and a positive halo sign. The client is in the intensive care unit, sedated and on a ventilator, and is in critical but stable condition. What collaborative problem takes priority at this time? a. Inability to communicate b. Nutritional deficit c. Risk for acquiring an infection d. Risk for skin breakdown

ANS: C The positive halo sign indicates a leak of cerebrospinal fluid. This places the client at high risk of acquiring an infection. Communication and nutrition are not priorities compared with preventing a brain infection. The client has a definite risk for a skin breakdown, but it is not the immediate danger a brain infection would be.

A client had an embolectomy for an arteriovenous malformation (AVM). The client is now reporting a severe headache and has vomited. What action by the nurse takes priority? a. Administer pain medication. b. Assess the client's vital signs. c. Notify the Rapid Response Team. d. Raise the head of the bed.

ANS: C This client may be experiencing a rebleed from the AVM. The most important action is to call the Rapid Response Team as this is an emergency. The nurse can assess vital signs while someone else notifies the Team, but getting immediate medical attention is the priority. Administering pain medication may not be warranted if the client must return to surgery. The optimal position for the client with an AVM has not been determined, but calling the Rapid Response Team takes priority over positioning.

The nurse assesses a client's Glasgow Coma Scale (GCS) score and determines it to be 12 (a 4 in each category). What care should the nurse anticipate for this client? a. Can ambulate independently b. May have trouble swallowing c. Needs frequent re-orientation d. Will need near-total care

ANS: C This client will most likely be confused and need frequent re-orientation. The client may not be able to ambulate at all but should do so independently, not because of mental status. Swallowing is not assessed with the GCS. The client will not need near-total care.

An intubated client's oxygen saturation has dropped to 88%. What action by the nurse takes priority? a. Determine if the tube is kinked. b. Ensure all connections are patent. c. Listen to the client's lung sounds. d. Suction the endotracheal tube.

ANS: C When an intubated client shows signs of hypoxia, check for DOPE: displaced tube (most common cause), obstruction (often by secretions), pneumothorax, and equipment problems. The nurse listens for equal, bilateral breath sounds first to determine if the endotracheal tube is still correctly placed. If this assessment is normal, the nurse would follow the mnemonic and assess the patency of the tube and connections and perform suction.

A nurse reviews laboratory results for a client with glomerulonephritis. The client's glomerular filtration rate (GFR) is 40 mL/min as measured by a 24-hour creatinine clearance. How should the nurse interpret this finding? (Select all that apply.) a. Excessive GFR b. Normal GFR c. Reduced GFR d. Potential for fluid overload e. Potential for dehydration

ANS: C, D The GFR refers to the initial amount of urine that the kidneys filter from the blood. In the healthy adult, the normal GFR ranges between 100 and 120 mL/min, most of which is reabsorbed in the kidney tubules. A GFR of 40 mL/min is drastically reduced, with the client experiencing fluid retention and risks for hypertension and pulmonary edema as a result of excess vascular fluid.

A nurse receives a report on a client who had a left-sided stroke and has homonymous hemianopsia. What action by the nurse is most appropriate for this client? a. Assess for bladder retention and/or incontinence. b. Listen to the client's lungs after eating or drinking. c. Prop the client's right side up when sitting in a chair. d. Rotate the client's meal tray when the client stops eating.

ANS: D This condition is blindness on the same side of both eyes. The client must turn his or her head to see the entire visual field. The client may not see all the food on the tray, so the nurse rotates it so uneaten food is now within the visual field. This condition is not related to bladder function, difficulty swallowing, or lack of trunk control.

The nurse is caring for four clients with traumatic brain injuries. Which client should the nurse assess first? a. Client with cerebral perfusion pressure of 72 mm Hg b. Client who has a Glasgow Coma Scale score of 12 c. Client with a PaCO2 of 36 mm Hg who is on a ventilator d. Client who has a temperature of 102° F (38.9° C)

ANS: D A fever is a poor prognostic indicator in clients with brain injuries. The nurse should see this client first. A Glasgow Coma Scale score of 12, a PaCO2 of 36, and cerebral perfusion pressure of 72 mm Hg are all desired outcomes.

A nurse teaches a client who has a history of heart failure. Which statement should the nurse include in this client's discharge teaching? a. "Avoid drinking more than 3 quarts of liquids each day." b. "Eat six small meals daily instead of three larger meals." c. "When you feel short of breath, take an additional diuretic." d. "Weigh yourself daily while wearing the same amount of clothing."

ANS: D Clients with heart failure are instructed to weigh themselves daily to detect worsening heart failure early, and thus avoid complications. Other signs of worsening heart failure include increasing dyspnea, exercise intolerance, cold symptoms, and nocturia. Fluid overload increases symptoms of heart failure. The client should be taught to eat a heart-healthy diet, balance intake and output to prevent dehydration and overload, and take medications as prescribed. The most important discharge teaching is daily weights as this provides the best data related to fluid retention.

To verify the correct placement of an endotracheal tube (ET) after insertion, the best initial action by the nurse is to a. auscultate for the presence of bilateral breath sounds. b. obtain a portable chest radiograph to check tube placement. c. observe the chest for symmetrical movement with ventilation. d. use an end-tidal CO2 monitor to check for placement in the trachea.

ANS: D End-tidal CO2 monitors are currently recommended for rapid verification of ET placement. Auscultation for bilateral breath sounds and checking chest expansion also are used, but they are not as accurate as end-tidal CO2 monitoring. A chest x-ray confirms the placement but is done after the tube is secured. DIF: Cognitive Level: Application REF: 1701-1702 TOP: Nursing Process: Evaluation MSC: NCLEX: Physiological Integrity

A nurse teaches a client who is prescribed digoxin (Lanoxin) therapy. Which statement should the nurse include in this client's teaching? a. "Avoid taking aspirin or aspirin-containing products." b. "Increase your intake of foods that are high in potassium." c. "Hold this medication if your pulse rate is below 80 beats/min." d. "Do not take this medication within 1 hour of taking an antacid."

ANS: D Gastrointestinal absorption of digoxin is erratic. Many medications, especially antacids, interfere with its absorption. Clients are taught to hold their digoxin for bradycardia; a heart rate of 80 beats/min is too high for this cutoff. Potassium and aspirin have no impact on digoxin absorption, nor do these statements decrease complications of digoxin therapy.

A client is assessed by the nurse after a hemodialysis session. The nurse notes bleeding from the client's nose and around the intravenous catheter. What action by the nurse is the priority? a. Hold pressure over the client's nose for 10 minutes. b. Take the client's pulse, blood pressure, and temperature. c. Assess for a bruit or thrill over the arteriovenous fistula. d. Prepare protamine sulfate for administration.

ANS: D Heparin is used with hemodialysis treatments. The bleeding alerts the nurse that too much anticoagulant is in the client's system and protamine sulfate should be administered. Pressure, taking vital signs, and assessing for a bruit or thrill are not as important as medication administration.

A patient with a subarachnoid hemorrhage is intubated and placed on a mechanical ventilator. When monitoring the patient, the nurse will need to notify the health care provider if the patient develops a. oxygen saturation of 94%. b. respirations of 18 breaths/min. c. green nasogastric tube drainage. d. increased jugular vein distention (JVD).

ANS: D Increases in JVD in a patient with a subarachnoid hemorrhage may indicate an increase in intra-cranial pressure (ICP) and that the PEEP setting is too high for this patient. A respiratory rate of 18, O2 saturation of 94%, and green nasogastric tube drainage are normal. DIF: Cognitive Level: Application REF: 1711-1712 OBJ: Special Questions: Prioritization TOP: Nursing Process: Assessment MSC: NCLEX: Physiological Integrity

A nurse cares for a client who is hemorrhaging from bleeding esophageal varices and has an esophagogastric tube. Which action should the nurse take first? a. Sedate the client to prevent tube dislodgement. b. Maintain balloon pressure at 15 and 20 mm Hg. c. Irrigate the gastric lumen with normal saline. d. Assess the client for airway patency.

ANS: D Maintaining airway patency is the primary nursing intervention for this client. The nurse suctions oral secretions to prevent aspiration and occlusion of the airway. The client usually is intubated and mechanically ventilated during this treatment. The client should be sedated, balloon pressure should be maintained between 15 and 20 mm Hg, and the lumen can be irrigated with saline or tap water. However, these are not a higher priority than airway patency.

A nurse assesses a client who is prescribed an infusion of vasopressin (Pitressin) for bleeding esophageal varices. Which clinical manifestation should alert the nurse to a serious adverse effect? a. Nausea and vomiting b. Frontal headache c. Vertigo and syncope d. Mid-sternal chest pain

ANS: D Mid-sternal chest pain is indicative of acute angina or myocardial infarction, which can be precipitated by vasopressin. Nausea and vomiting, headache, and vertigo and syncope are not side effects of vasopressin.

A client undergoing hemodynamic monitoring after a myocardial infarction has a right atrial pressure of 0.5 mm Hg. What action by the nurse is most appropriate? a. Level the transducer at the phlebostatic axis. b. Lay the client in the supine position. c. Prepare to administer diuretics. d. Prepare to administer a fluid bolus.

ANS: D Normal right atrial pressures are from 1 to 8 mm Hg. Lower pressures usually indicate hypovolemia, so the nurse should prepare to administer a fluid bolus. The transducer should remain leveled at the phlebostatic axis. Positioning may or may not influence readings. Diuretics would be contraindicated.

A nurse is caring for a client with acute pericarditis who reports substernal precordial pain that radiates to the left side of the neck. Which nonpharmacologic comfort measure should the nurse implement? a. Apply an ice pack to the client's chest. b. Provide a neck rub, especially on the left side. c. Allow the client to lie in bed with the lights down. d. Sit the client up with a pillow to lean forward on.

ANS: D Pain from acute pericarditis may worsen when the client lays supine. The nurse should position the client in a comfortable position, which usually is upright and leaning slightly forward. Pain is decreased by using gravity to take pressure off the heart muscle. An ice pack and neck rub will not relieve this pain.

A client has just had a central line catheter placed that is specific for hemodialysis. What is the most appropriate action by the nurse? a. Use the catheter for the next laboratory blood draw. b. Monitor the central venous pressure through this line. c. Access the line for the next intravenous medication. d. Place a heparin or heparin/saline dwell after hemodialysis.

ANS: D The central line should have a heparin or heparin/saline dwell after hemodialysis treatment. The central line catheter used for dialysis should not be used for blood sampling, monitoring central venous pressures, or giving drugs or fluids.

A nurse cares for a client who is recovering after a nephrostomy tube was placed 6 hours ago. The nurse notes drainage in the tube has decreased from 40 mL/hr to 12 mL over the last hour. Which action should the nurse take? a. Document the finding in the client's record. b. Evaluate the tube as working in the hand-off report. c. Clamp the tube in preparation for removing it. d. Assess the client's abdomen and vital signs.

ANS: D The nephrostomy tube should continue to have a consistent amount of drainage. If the drainage slows or stops, it may be obstructed. The nurse must notify the provider, but first should carefully assess the client's abdomen for pain and distention and check vital signs so that this information can be reported as well. The other interventions are not appropriate.

A client with acute kidney injury has a blood pressure of 76/55 mm Hg. The health care provider ordered 1000 mL of normal saline to be infused over 1 hour to maintain perfusion. The client is starting to develop shortness of breath. What is the nurse's priority action? a. Calculate the mean arterial pressure (MAP). b. Ask for insertion of a pulmonary artery catheter. c. Take the client's pulse. d. Slow down the normal saline infusion.

ANS: D The nurse should assess that the client could be developing fluid overload and respiratory distress and slow down the normal saline infusion. The calculation of the MAP also reflects perfusion. The insertion of a pulmonary artery catheter would evaluate the client's hemodynamic status, but this should not be the initial action by the nurse. Vital signs are also important after adjusting the intravenous infusion.

When the ventilator alarm sounds, the nurse finds the patient lying in bed holding the endotracheal tube (ET). Which action should the nurse take first? a. Offer reassurance to the patient. b. Activate the hospital's rapid response team. c. Call the health care provider to reinsert the tube. d. Manually ventilate the patient with 100% oxygen.

ANS: D The nurse should ensure maximal patient oxygenation by manually ventilating with a bag-valve-mask system. Offering reassurance to the patient, notifying the health care provider about the need to reinsert the tube, and activating the rapid response team also are appropriate after the nurse has stabilized the patient's oxygenation. DIF: Cognitive Level: Application REF: 1704-1706 OBJ: Special Questions: Prioritization TOP: Nursing Process: Implementation MSC: NCLEX: Physiological Integrity

An hour after a thoracotomy, a patient complains of incisional pain at a level 7 (based on 0 to 10 scale) and has decreased left-sided breath sounds. The pleural drainage system has 100 mL of bloody drainage and a large air leak. Which action is best for the nurse to take next? a. Milk the chest tube gently to remove any clots. b. Clamp the chest tube momentarily to check for the origin of the air leak. c. Assist the patient to deep breathe, cough, and use the incentive spirometer. d. Set up the patient controlled analgesia (PCA) and administer the loading dose of morphine.

ANS: D The patient is unlikely to take deep breaths or cough until the pain level is lower. A chest tube output of 100 mL is not unusual in the first hour after thoracotomy and would not require milking of the chest tube. An air leak is expected in the initial postoperative period after thoracotomy.

When assessing a patient who has just arrived after an automobile accident, the emergency department nurse notes tachycardia and absent breath sounds over the right lung. For which intervention will the nurse prepare the patient? a. Emergency pericardiocentesis b. Stabilization of the chest wall with tape c. Administration of an inhaled bronchodilator d. Insertion of a chest tube with a chest drainage system

ANS: D The patient's history and absent breath sounds suggest a right-sided pneumothorax or hemothorax, which will require treatment with a chest tube and drainage. The other therapies would be appropriate for an acute asthma attack, flail chest, or cardiac tamponade, but the patient's clinical manifestations are not consistent with these problems.

After change-of-shift report, which patient should the nurse assess first? a. 72-year-old with cor pulmonale who has 4+ bilateral edema in his legs and feet b. 28-year-old with a history of a lung transplant and a temperature of 101° F (38.3° C) c. 40-year-old with a pleural effusion who is complaining of severe stabbing chest pain d. 64-year-old with lung cancer and tracheal deviation after subclavian catheter insertion

ANS: D The patient's history and symptoms suggest possible tension pneumothorax, a medical emergency. The other patients also require assessment as soon as possible, but tension pneumothorax will require immediate treatment to avoid death from inadequate cardiac output or hypoxemia.

A client with a stroke is being evaluated for fibrinolytic therapy. What information from the client or family is most important for the nurse to obtain? a. Loss of bladder control b. Other medical conditions c. Progression of symptoms d. Time of symptom onset

ANS: D The time limit for initiating fibrinolytic therapy for a stroke is 3 to 4.5 hours, so the exact time of symptom onset is the most important information for this client. The other information is not as critical.

A client is brought to the emergency department after sustaining injuries in a severe car crash. The client's chest wall does not appear to be moving normally with respirations, oxygen saturation is 82%, and the client is cyanotic. What action by the nurse is the priority? a. Administer oxygen and reassess. b. Auscultate the client's lung sounds. c. Facilitate a portable chest x-ray. d. Prepare to assist with intubation.

ANS: D This client has manifestations of flail chest and, with the other signs, needs to be intubated and mechanically ventilated immediately. The nurse does not have time to administer oxygen and wait to reassess, or to listen to lung sounds. A chest x-ray will be taken after the client is intubated.

A client in the intensive care unit is scheduled for a lumbar puncture (LP) today. On assessment, the nurse finds the client breathing irregularly with one pupil fixed and dilated. What action by the nurse is best? a. Ensure that informed consent is on the chart. b. Document these findings in the client's record. c. Give the prescribed preprocedure sedation. d. Notify the provider of the findings immediately.

ANS: D This client is exhibiting signs of increased intracranial pressure. The nurse should notify the provider immediately because performing the LP now could lead to herniation. Informed consent is needed for an LP, but this is not the priority. Documentation should be thorough, but again this is not the priority. The preprocedure sedation (or other preprocedure medications) should not be given as the LP will most likely be canceled.

The nurse is preparing to change a client's sternal dressing. What action by the nurse is most important? a. Assess vital signs. b. Don a mask and gown. c. Gather needed supplies. d. Perform hand hygiene.

ANS: D To prevent a sternal wound infection, the nurse washes hands or performs hand hygiene as a priority. Vital signs do not necessarily need to be assessed beforehand. A mask and gown are not needed. The nurse should gather needed supplies, but this is not the priority.

An older adult pt presents to the ED with a 4-day history of cough, SOB, pain on inspiration, and dyspnea. The pt never had a pneumococcal vaccine. The chest x-ray shows density in bilateral lung bases. The pt has wheezing upon auscultation of both lungs. Would a bronchodilator be beneficial for this pt? A. It would not be beneficial for this pt. B. It would help decrease the bronchospasm. C. It would clear up the density in the pts lung bases. D. The pt would have a decrease in the pain on inspiration.

B A bronchodilator would would open up the airways and help decrease bronchospasm , so it would be beneficial for this pt. It would decrease dyspnea and feelings of shortness of breath.

INH treatment is associated with the development of peripheral neuropathies. Which of the following interventions would the nurse teach the client to help prevent this complication? A. Adhere to a low cholesterol diet B. Supplement the diet with pyridoxine (vitamin B6) C. Get extra rest D. Avoid excessive sun exposure

B INH competes with the available vitamin B6 in the body and leaves the client at risk for development of neuropathies related to vitamin deficiency. Supplemental vitamin B6 is routinely prescribed.

Which symptom of pneumonia may present differently in the older adult pt than in the younger adult pt? A. Crackles on auscultation B. Fever C. Headache D. Wheezing

B Older adults may not have fever and may have a lower-than-normal temperature with pneumonia. The other answers may be seen in all age groups of pneumonia pts

Which statement best describes pneumonia? A. an infection of just the windpipe because the lungs are clear of any problems B. a serious inflammation, caused by various things, of the bronchioles C. only an infection of the lungs with mild to severe effects on breathing D. an inflammation resulting from damage to the lungs due to long-term smoking

B a serious inflammation, caused by various things, of the bronchioles

The pt with pneumonia has a priority problem of ineffective airway clearance with bronchospasms. Pt has no previous chronic resp disorders. The nurse will obtain an order for which intervention? A. increased liters of humidified oxygen via facemask B. scheduled and prn aerosol nebulizer bronchodilator treatments C. handheld bronchodilator inhaler prn D. corticosteroid via inhaler or IV to reduce inflammation

B scheduled and prn aerosol nebulizer bronchodilator treatments

You are caring for a patient admitted with a subdural hematoma after a motor vehicle accident. Which change in vital signs would you interpret as a manifestation of increased intracranial pressure? A. Tachypnea B. Bradycardia C. Hypotension D. Narrowing pulse pressure

B Changes in vital signs indicative of increased ICP are known as Cushing's triad, which consists of increasing systolic pressure with a widening pulse pressure, bradycardia with a full and bounding pulse, and irregular respirations.

The home health nurse is visiting the home of an older adult pt recovering from a knee replacement. She ids a priority pt problem of risk for respiratory infection. Which is a normal aging factor contributing to the risk A. inability of a forced cough B. decreased strength of resp. muscles C. increased macrophages in alveoli D. increased elastic recoil of alveoli

B decr strength resp muscles

A nurse is triaging clients in the emergency department (ED). Which client should the nurse prioritize to receive care first? a. A 22-year-old with a painful and swollen right wrist b. A 45-year-old reporting chest pain and diaphoresis c. A 60-year-old reporting difficulty swallowing and nausea d. An 81-year-old with a respiratory rate of 28 breaths/min and a temperature of 101 F

B ~ A client experiencing chest pain and diaphoresis would be classified as emergent and would be triaged immediately to a treatment room in the ED. The other clients are more stable.

A nurse is evaluating levels and functions of trauma centers. Which function is appropriately paired with the level of the trauma center? a. Level I Located within remote areas and provides advanced life support within resource capabilities b. Level II Located within community hospitals and provides care to most injured clients c. Level III Located in rural communities and provides only basic care to clients d. Level IV Located in large teaching hospitals and provides a full continuum of trauma care for all clients

B ~ Level I trauma centers are usually located in large teaching hospital systems and provide a full continuum of trauma care for all clients. Both Level II and Level III facilities are usually located in community hospitals. These trauma centers provide care for most clients and transport to Level I centers when client needs exceed resource capabilities. Level IV trauma centers are usually located in rural and remote areas. These centers provide basic care, stabilization, and advanced life support while transfer arrangements to higher-level trauma centers are made.

A trauma client with multiple open wounds is brought to the emergency department in cardiac arrest. Which action should the nurse take prior to providing advanced cardiac life support? a. Contact the on-call orthopedic surgeon. b. Don personal protective equipment. c. Notify the Rapid Response Team. d. Obtain a complete history from the paramedic.

B ~ Nurses must recognize and plan for a high risk of contamination with blood and body fluids when engaging in trauma resuscitation. Standard Precautions should be taken in all resuscitation situations and at other times when exposure to blood and body fluids is likely. Proper attire consists of an impervious cover gown, gloves, eye protection, a facemask, a surgical cap, and shoe covers.

A client with pneumonia caused by aspiration after alcohol intoxication has just been admitted. The client is febrile and agitated. Which health care provider order should the nurse implement first? A) Administer levofloxacin (Levaquin) 500 mg IV. B) Draw aerobic and anaerobic blood cultures. C) Give lorazepam (Ativan) as needed for agitation. D) Refer to social worker for alcohol counseling.

B) Draw aerobic and anaerobic blood cultures.

pneumonia may present differently in the older adult than in the younger adult? A) Crackles on auscultation B) Fever C) Headache D) Wheezing

B) Fever

The critical care nurse is caring for a patient who is in cardiogenic shock. What assessments must the nurse perform on this patient? Select all that apply. A) Platelet level B) Fluid status C) Cardiac rhythm D) Action of medications E) Sputum volume

B) Fluid status C) Cardiac rhythm D) Action of medications The critical care nurse must carefully assess the patient in cardiogenic shock, observe the cardiac rhythm, monitor hemodynamic parameters, monitor fluid status, and adjust medications and therapies based on the assessment data. Platelet levels and sputum production are not major assessment parameters in a patient who is experiencing cardiogenic shock.

An older client presents to the emergency department with a 2-day history of cough, pain on inspiration, shortness of breath, and dyspnea. The client never had a pneumococcal vaccine. The client's chest x-ray shows density in both bases. The client has wheezing upon auscultation of both lungs. Would a bronchodilator be beneficial for this client? A) It would not be beneficial for this client. B) It would help decrease the bronchospasm. C) It would clear up the density in the bases of the client's lungs. D) It would decrease the client's pain on inspiration.

B) It would help decrease the bronchospasm.

A nurse is caring for clients in a busy emergency department. Which actions should the nurse take to ensure client and staff safety? (SATA) a. Leave the stretcher in the lowest position with rails down so that the client can access the bathroom. b. Use two identifiers before each intervention and before mediation administration. c. Attempt de-escalation strategies for clients who demonstrate aggressive behaviors. d. Search the belongings of clients with altered mental status to gain essential medical information. e. Isolate clients who have immune suppression disorders to prevent hospital-acquired infections.

B, C, D ~ To ensure client and staff safety, nurses should use two identifiers per The Joint Commissions National Patient Safety Goals; follow the hospitals security plan, including de-escalation strategies for people who demonstrate aggressive or violent tendencies; and search belongings to identify essential medical information. Nurses should also use standard fall prevention interventions, including leaving stretchers in the lowest position with rails up, and isolating clients who present with signs and symptoms of contagious infectious disorders.

An emergency room nurse is caring for a trauma client. Which interventions should the nurse perform during the primary survey? (SATA) a. Foley catheterization b. Needle decompression c. Initiating IV fluids d. Splinting open fractures e. Endotracheal intubation f. Removing wet clothing g. Laceration repair

B, C, E, F ~ The primary survey for a trauma client organizes the approach to the client so that life-threatening injuries are rapidly identified and managed. The primary survey is based on the standard mnemonic ABC, with an added D and E: Airway and cervical spine control; Breathing; Circulation; Disability; and Exposure. After the completion of primary diagnostic and laboratory studies, and the insertion of gastric and urinary tubes, the secondary survey (a complete head-to-toe assessment) can be carried out.

Which patient is most likely going into respiratory failure? A. A patient who report that he feels short of breath while eating B. A patient with the following arterial blood gas values over the past 3 hours: pH 7.50, 7.45, and 7.40 C. A patient with an oxygen saturation value of 93% D. A patient with chronic obstructive pulmonary disease (COPD) who has distant breath sounds

B. A patient with the following arterial blood gas values over the past 3 hours: pH 7.50, 7.45, and 7.40 Manifestations of respiratory failure are related to the extent of change in PaO2 or PaCO2, the rapidity of change, and ability to compensate. It is important to monitor trends. Shortness of breath is a subjective report, and it can have many causes. A single borderline oxygen saturation reading is not as indicative of failure as a negative trend. Because of air trapping with COPD, the breath sounds are typically distant.

The nurse recognizes the presence of Cushing's triad in the patient with a. Increased pulse, irregular respiration, increased BP b. decreased pulse, irregular respiration, increased pulse pressure c. increased pulse, decreased respiration, increased pulse pressure d. decreased pulse, increased respiration, decreased systolic BP

B. Cushing's triad consists of three vital sign measures that reflect ICP and its effect on the medulla, the hypothalamus, the pons, and the thalamus. Because these structures are very deep, Cushing's triad is usually a late sign of ICP. The signs include an increasing systolic BP with a widening pulse pressure, a bradycardia with a full and bounding pulse, and irregular respirations.

A patient in the clinic reports a recent episode of dysphasia and left-sided weakness at home that resolved after 2 hours. The nurse will anticipate teaching the patient about a. alteplase (tPA). b. aspirin (Ecotrin). c. warfarin (Coumadin). d. nimodipine (Nimotop).

B. Following a transient ischemic attack (TIA), patients typically are started on medications such as aspirin to inhibit platelet function and decrease stroke risk. tPA is used for acute ischemic stroke. Coumadin is usually used for patients with atrial fibrillation. Nimodipine is used to prevent cerebral vasospasm after a subarachnoid hemorrhage.

Which of the following additional assessment data should immediately be gathered to determine the status of a client with a RR of 4 bpm? A. Arterial blood gas (ABG) & breath sounds B. Level of consciousness & pulse oximetry value C. Breath sounds & reflexes D. Pulse oximetry value & heart sounds

B. Level of consciousness & pulse oximetry value First, the nurse should attempt to arouse the client because this should increase the client's RR.

Which is a proper nursing action for a patient in acute respiratory failure? A. Administer 100% oxygen to an intubated patient until the pathology has resolved. B. Provide chest physical therapy for patients who produce more than 30 mL of sputum per day. C. Use continuous positive airway pressure (CPAP) if the patient has weak or absent respirations. D. Administer packed red blood cells to maintain the hemoglobin level at 7 g/dL or higher.

B. Provide chest physical therapy for patients who produce more than 30 mL of sputum per day. Chest physical therapy is indicated for patients who produce more than 30 mL of sputum per day or have evidence of atelectasis or pulmonary infiltrates. The selected oxygen delivery system must also maintain PaO2 equal to or more than 55 to 60 mm HG and SaO2 equal or greater than 90% at the lowest O2 concentration possible. High oxygen concentrations replace the nitrogen gas normally present in the alveoli, causing instability and atelectasis. In intubated patients, exposure to 60% or more oxygen for longer than 48 hours poses a significant risk for oxygen toxicity. Noninvasive positive-pressure ventilation such as CPAP is not appropriate for patients who have weak or no respirations (are not inhaling). The hemoglobin level should be equal to or greater than 9 g/dL to ensure adequate oxygen saturation.

A patient with a head injury has bloody drainage from the ear. To determine whether CSF is present in the drainage, the nurse a. examines the tympanic membrane for a tear b. tests the fluid for a halo sign on a white dressing c. tests the fluid with a glucose identifying strip or stick d. collects 5 mL of fluid in a test tube and sends it to the laboratory for analysis

B. Tests the fluid for a halo sing on a white dressing- Testing clear drainage for CSF in nasal or ear drainage may be done with a Dextrostik or Tes-Tape strip, but if blood is present, the glucose in the blood will produce and unreliable result. To test bloody drainage, the nurse should test the fluid for a halo or ring that occurs when a yellowish ring encircles blood dripped onto a white pad or towel

Which intervention is most likely to prevent or limit barotrauma in the patient with ARDS who is mechanically ventilated? A. Decreasing PEEP B. Use of permissive hypercapnia C. Increasing the tidal volume D. Use of positive pressure ventilation

B. Use of permissive hypercapnia To avoid barotrauma and minimize risks associated with elevated plateau and peak inspiratory pressures, the patient with ARDS is often ventilated with smaller tidal volumes and various amounts of PEEP to minimize oxygen requirements and intrathoracic pressures. One result of this protocol is an elevation in PaCO2, also called permissive hypercapnia because the PaCO2 is allowed to rise above normal limits.

The nurse plans care for a patient with increased ICP with the knowledge that the best way to position the patient is to a. keep the head of the bed flat b. elevate the head of the bed to 30 degrees c. maintain patient on the left side with the head supported on a pillow d. use a continuous rotation bed to continuously change patient position

B. elevate the head of the bed to 30 degrees

A patient with increased ICP has mannitol (Osmitrol) prescribed. Which option is the best indication that the drug is achieving the desired therapeutic effects? A. Urine output increases from 30 mL to 50 mL/hour. B. Blood pressure remains less than 150/90 mm Hg. C. The LOC improves. D. No crackles are auscultated in the lung fields.

C LOC is the most sensitive indicator of ICP. Mannitol is an osmotic diuretic that works to decrease the ICP by plasma expansion and an osmotic effect. Although the other options may indicate a therapeutic effect of a diuretic, they are not the main reason this drug is given.

A pt is admitted to the hospital with bronchopneumonia. The nurse knows that this pt has pneumonia that A. has only affected a certain lobe of the lung B. has affected bilateral lower lobes C. is scattered throughout the lung, with affected patches throughout. D. will cause aspiration, so pt should be monitored

C bronchopneumonia is scattered affected areas throughout multiple lobes of the lungs A. = a definition of lobar pneumonia

A nurse is triaging clients in the emergency department. Which client should the nurse classify as nonurgent? a. A 44-year-old with chest pain and diaphoresis b. A 50-year-old with chest trauma and absent breath sounds c. A 62-year-old with a simple fracture of the left arm d. A 79-year-old with a temperature of 104 F

C ~ A client in a nonurgent category can tolerate waiting several hours for health care services without a significant risk of clinical deterioration. The client with a simple arm fracture and palpable radial pulses is currently stable, is not at significant risk of clinical deterioration, and would be considered nonurgent. The client with chest pain and diaphoresis and the client with chest trauma are emergent owing to the potential for clinical deterioration and would be seen immediately. The client with a high fever may be stable now but also has a risk of deterioration.

A nurse is triaging clients in the emergency department. Which client should be considered urgent? a. A 20-year-old female with a chest stab wound and tachycardia b. A 45-year-old homeless man with a skin rash and sore throat c. A 75-year-old female with a cough and a temperature of 102 F d. A 50-year-old male with new-onset confusion and slurred speech

C ~ A client with a cough and a temperature of 102 F is urgent. This client is at risk for deterioration and needs to be seen quickly, but is not in an immediately life-threatening situation. The client with a chest stab wound and tachycardia and the client with new-onset confusion and slurred speech should be triaged as emergent. The client with a skin rash and a sore throat is not at risk for deterioration and would be triaged as nonurgent.

While triaging clients in a crowded emergency department, a nurse assesses a client who presents with symptoms of tuberculosis. Which action should the nurse take first? a. Apply oxygen via nasal cannula. b. Administer intravenous 0.9% saline solution. c. Transfer the client to a negative-pressure room. d. Obtain a sputum culture and sensitivity.

C ~ A client with signs and symptoms of tuberculosis or other airborne pathogens should be placed in a negative-pressure room to prevent contamination of staff, clients & family members in the crowded emergency department.

An emergency room nurse assesses a client who has been raped. With which health care team member should the nurse collaborate when planning this client's care? a. Emergency medicine physician b. Case manager c. Forensic nurse examiner d. Psychiatric crisis nurse

C ~ All other members of the health care team listed may be used in the management of this client's care. However, the forensic nurse examiner is educated to obtain client histories and collect evidence dealing with the assault, and can offer the counseling and follow-up needed when dealing with the victim of an assault.

An emergency room nurse is triaging victims of a multi-casualty event. Which client should receive care first? a. A 30-year-old distraught mother holding her crying child b. A 65-year-old conscious male with a head laceration c. A 26-year-old male who has pale, cool, clammy skin d. A 48-year-old with a simple fracture of the lower leg

C ~ The client with pale, cool, clammy skin is in shock and needs immediate medical attention. The mother does not have injuries and so would be the lowest priority. The other two people need medical attention soon, but not at the expense of a person in shock.

An emergency department nurse is caring for a client who is homeless. Which action should the nurse take to gain the clients trust? a. Speak in a quiet and monotone voice. b. Avoid eye contact with the client. c. Listen to the client's concerns and needs. d. Ask security to store the client's belongings.

C ~ To demonstrate behaviors that promote trust with homeless clients, the emergency room nurse should make eye contact (if culturally appropriate), speak calmly, avoid any prejudicial or stereotypical remarks, show genuine care and concern by listening, and follow through on promises. The nurse should also respect the client's belongings and personal space.

You are aware of the value of using a mini-tracheostomy to facilitate suctioning when patients are unable to independently mobilize their secretions. For which patient is the use of a mini-tracheostomy indicated? A. A patient whose recent ischemic stroke has resulted in the loss of his gag reflex B. A patient who requires long-term mechanical ventilation as the result of a spinal cord injury C. A patient whose increased secretions are the result of community-acquired pneumonia D. A patient with a head injury who has developed aspiration pneumonia

C. A patient whose increased secretions are the result of community-acquired pneumonia It is probably appropriate to suction a patient with pneumonia using a mini-tracheostomy if blind suctioning is ineffective or difficult. An absent or compromised gag reflex, long-term ventilation, and a history of aspiration contraindicates the use of a mini-tracheostomy.

You are admitting a 45-year-old asthmatic patient in acute respiratory distress. You auscultate the patient's lungs and notice cessation of inspiratory wheezing. The patient has not yet received any medication. What does this finding suggest? A. Spontaneous resolution of the acute asthma attack B. An acute development of bilateral pleural effusions C. Airway constriction requiring intensive interventions D. Overworked intercostal muscles resulting in poor air exchange

C. Airway constriction requiring intensive interventions When the patient in respiratory distress has inspiratory wheezing that ceases, it is an indication of airway obstruction, and it requires emergency action to restore the airway.

A 72-year-old patient who has a history of a transient ischemic attack (TIA) has an order for aspirin 160 mg daily. When the nurse is administering medications, the patient says, "I don't need the aspirin today. I don't have a fever." Which action should the nurse take? a. Document that the aspirin was refused by the patient. b. Tell the patient that the aspirin is used to prevent a fever. c. Explain that the aspirin is ordered to decrease stroke risk. d. Call the health care provider to clarify the medication order.

C. Aspirin is ordered to prevent stroke in patients who have experienced TIAs. Documentation of the patient's refusal to take the medication is an inadequate response by the nurse. There is no need to clarify the order with the health care provider. The aspirin is not ordered to prevent aches and pains.

A 58 year old client with a 40 year history of smoking one to two packs of cigarettes a day has a chronic cough producing thick sputum, peripheral edema, & cyanotic nail beds. Based on this information, he most likely has which of the following conditions? A. Adult respiratory distress syndrome (ARDS) B. Asthma C. Chronic obstructive bronchitis D. Emphysema

C. Chronic obstructive bronchitis Because of his extensive smoking history & symptoms, the client most likely has chronic obstructive bronchitis. Clients with asthma & emphysema tend not to have a chronic cough or peripheral edema.

The term "blue bloater" describes which condition? A. Adult respiratory distress syndrome (ARDS) B. Asthma C. Chronic obstructive bronchitis D. Emphysema

C. Chronic obstructive bronchitis Clients with this appear bloated. They have large barrel chests, peripheral edema, cyanotic nail beds, & at times circumoral cyanosis. Clients with emphysema appear pink & cachectic.

You are caring for a patient who is admitted with a barbiturate overdose. The patient is unresponsive, with a blood pressure of 90/60 mm Hg, apical pulse of 110 beats/minute, and respiratory rate of 8 breaths/minute. Based on the initial assessment findings, you recognize that the patient is at risk for which type of respiratory failure? A. Hypoxemic respiratory failure related to shunting of blood B. Hypoxemic respiratory failure related to diffusion limitation C. Hypercapnic respiratory failure related to alveolar hypoventilation D. Hypercapnic respiratory failure related to increased airway resistance

C. Hypercapnic respiratory failure related to alveolar hypoventilation The patient's respiratory rate is decreased because of barbiturate overdose, which causes respiratory depression. The patient is at risk for hypercapnic respiratory failure resulting from the decreased respiratory rate and decreased CO2 exchange.

Which of the following assessment findings would help confirm a diagnosis of asthma in a client suspected of having the disorder? A. Circumoral cyanosis B. Increased forced expiratory volume C. Inspiratory & expiratory wheezing D. Normal breath sounds

C. Inspiratory & expiratory wheezing

The earliest signs of increased ICP the nurse should assess for include a. Cushing's triad b. unexpected vomiting c. decreasing level of consciousness (LOC) d. dilated pupil with sluggish response to light

C. One of the most sensitive signs of increased intracranial pressure (ICP) is a decreasing LOC. A decrease in LOC will occur before changes in vital signs, ocular signs, and projectile vomiting occur

While the nurse performs ROM on an unconscious patient with increased ICP, the patient experiences severe decerebrate posturing reflexes. The nurse should a. use restraints to protect the patient from injury b. administer CNS depressants to lightly sedate the patient c. perform the exercises less frequently because posturing can increase ICP d. continue the exercises because they are necessary to maintain musculoskeletal function

C. Perform the exercises less frequently because posturing can increase ICP- If reflex posturing occurs during ROM or positioning of the patient, these activities should be done less frequently until the patient's condition stabilizes, because posturing can case increases in ICP. Neither restraints nor CNS depressants would be indicated.

Which intervention is key to preventing ventilator-associated pneumonia as a complication in a patient with acute respiratory distress syndrome (ARDS)? A. Scheduled prophylactic nasopharyngeal suctioning B. Instilling normal saline down the endotracheal tube to loosen secretions C. Providing frequent mouth care and oral hygiene D. Using high tidal volumes on the ventilator

C. Providing frequent mouth care and oral hygiene A frequent complication of ARDS is ventilator-associated pneumonia. Preventative strategies include elevating head-of-bed 30-45 degrees and strict infection control measures such as frequent hand washing, use of in-line suction, and frequent mouth care and oral hygiene. Suctioning is done only as needed to prevent stimulating excess secretions. Instilling normal saline does not loosen secretions and can cause hypoxia. It is not recommended. High tidal volumes can lead to barotrauma.

A client has started a new drug for hypertension. Thirty minutes after he takes the drug, he develops chest tightness & becomes short of breath & tachypneic. He has decreased level of consciousness. These signs indicate which of the following conditions? A. Asthma attack B. Pulmonary embolism C. Respiratory failure D. Rheumatoid arthritis

C. Respiratory failure The client was reacting to the drug with respiratory signs impending anaphylaxis, which could lead to eventual respiratory failure.

A client with emphysema should receive only 1 to 3 L/minute of oxygen, if needed, or he may lose his hypoxic drive. Which of the following statements is correct about hypoxic drive? A. The client doesn't notice he needs to breathe B. The client breathes only when his oxygen levels climb above a certain point C. The client breathes only when his oxygen levels dip below a certain point D. The client breathes only when his carbon dioxide levels dip below a certain point

C. The client breathes only when his oxygen levels dip below a certain point Clients with emphysema breathe when their oxygen levels drop to a certain level; this is known as the hypoxic drive

Which is part of the nursing management for ARDS? A. Aggressive use of intravenous (IV) fluids B. Administration of a β-blocker C. Use of positive end-expiratory pressure (PEEP) D. Use of the lateral recumbent position

C. Use of positive end-expiratory pressure (PEEP) In ARDS, higher levels of PEEP may be used. It increases the functional residual capacity (FRC) and opens collapsed alveoli. The issues in ARDS treatment are respiratory related, not fluid deficit. β-Blockers are part of myocardial infarction management, not ARDS. Some ARDS patients do better when placed in a prone position instead of a supine position. In the supine position, the heart places pressure on the pleural cavity. Changing the patient to a prone position allows air-filled, nonatelectatic alveoli in the ventral portion of the lung to become dependent.

During the change of shift report a nurse is told that a patient has an occluded left posterior cerebral artery. The nurse will anticipate that the patient may have a. dysphasia. b. confusion. c. visual deficits. d. poor judgment.

C. Visual disturbances are expected with posterior cerebral artery occlusion. Aphasia occurs with middle cerebral artery involvement. Cognitive deficits and changes in judgment are more typical of anterior cerebral artery occlusion.

Maintenance of fluid balance in the patient with ARDS involves A. hydration using colloids. B. administration of surfactant. C. mild fluid restriction and diuretics as necessary. D. keeping the hemoglobin level greater than 12 g/dL (120 g/L).

C. mild fluid restriction and diuretics as necessary. Fluid balance in the patient with acute respiratory distress syndrome includes maintaining the pulmonary artery wedge pressure as low as possible without impairing cardiac output to limit pulmonary edema. The patient is usually placed on mild fluid restriction, and diuretics are used as necessary.

When caring for a patient who has had a head injury, which assessment information is of most concern to the nurse? a. The blood pressure increases from 120/54 to 136/62. b. The patient is more difficult to arouse. c. The patient complains of a headache at pain level 5 of a 10-point scale. d. The patient's apical pulse is slightly irregular.

Correct Answer: B Rationale: The change in level of consciousness (LOC) is an indicator of increased ICP and suggests that action by the nurse is needed to prevent complications. The change in BP should be monitored but is not an indicator of a need for immediate nursing action. Headache is not unusual in a patient after a head injury. A slightly irregular apical pulse is not unusual. Cognitive Level: Application Text Reference: p. 1470 Nursing Process: Assessment NCLEX: Physiological Integrity

A patient with possible cerebral edema has a serum sodium level of 115 mEq/L (115 mmol/L), a decreasing level of consciousness (LOC) and complains of a headache. All of the following orders have been received. Which one should the nurse accomplish first? a. Administer acetaminophen (Tylenol) 650 mg orally. b. Administer 5% hypertonic saline intravenously. c. Draw blood for arterial blood gases (ABGs). d. Send patient to radiology for computed tomography (CT) of the head.

Correct Answer: B Rationale: The patient's low sodium indicates that hyponatremia may be causing the cerebral edema, and the nurse's first action should be to correct the low sodium level. Acetaminophen (Tylenol) will have minimal effect on the headache because it is caused by cerebral edema and increased ICP. Drawing ABGs and obtaining a CT scan may add some useful information, but the low sodium level may lead to seizures unless it is addressed quickly. Cognitive Level: Application Text Reference: p. 1470 Nursing Process: Implementation NCLEX: Physiological Integrity

When assessing a patient with a head injury, the nurse recognizes that the earliest indication of increased intracranial pressure (ICP) is a. vomiting. b. headache. c. change in level of consciousness (LOC). d. sluggish pupil response to light.

Correct Answer: C Rationale: LOC is the most sensitive indicator of the patient's neurologic status and possible changes in ICP. Vomiting and sluggish pupil response to light are later signs of increased ICP. A headache can be caused by compression of intracranial structures as the brain swells, but it is not unexpected after a head injury. Cognitive Level: Comprehension Text Reference: p. 1470 Nursing Process: Assessment NCLEX: Physiological Integrity

A patient admitted with a head injury has admission vital signs of temperature 98.6° F (37° C), blood pressure 128/68, pulse 110, and respirations 26. Which of these vital signs, if taken 1 hour after admission, will be of most concern to the nurse? a. Blood pressure 130/72, pulse 90, respirations 32 b. Blood pressure 148/78, pulse 112, respirations 28 c. Blood pressure 156/60, pulse 60, respirations 14 d. Blood pressure 110/70, pulse 120, respirations 30

Correct Answer: C Rationale: Systolic hypertension with widening pulse pressure, bradycardia, and respiratory changes represent Cushing's triad and indicate that the ICP has increased and brain herniation may be imminent unless immediate action is taken to reduce ICP. The other vital signs may indicate the need for changes in treatment, but they are not indicative of an immediately life-threatening process. Cognitive Level: Application Text Reference: p. 1469 Nursing Process: Assessment NCLEX: Physiological Integrity

A patient with a head injury has an arterial blood pressure is 92/50 mm Hg and an intracranial pressure of 18 mm Hg. Which action by the nurse is appropriate? a. Document and continue to monitor the parameters. b. Elevate the head of the patient's bed. c. Notify the health care provider about the assessments. d. Check the patient's pupillary response to light.

Correct Answer: C Rationale: The patient's cerebral perfusion pressure is only 46 mm Hg, which will rapidly lead to cerebral ischemia and neuronal death unless rapid action is taken to reduce ICP and increase arterial BP. Documentation and monitoring are inadequate responses to the patient's problem. Elevating the head of the bed will lower the ICP but may also lower cerebral blood flow and further decrease CPP. Changes in pupil response to light are signs of increased ICP, so the nurse will only take more time doing this without adding any useful information. Cognitive Level: Analysis Text Reference: pp. 1468-1469 Nursing Process: Implementation NCLEX: Physiological Integrity

Mechanical ventilation with a rate and volume to maintain a mild hyperventilation is used for a patient with a head injury. To evaluate the effectiveness of the therapy, the nurse should a. monitor oxygen saturation. b. check arterial blood gases (ABGs). c. monitor intracranial pressure (ICP). d. assess patient breath sounds.

Correct Answer: C Rationale: The purpose of hyperventilation for a patient with a head injury is reduction of ICP, and ICP should be monitored to evaluate whether the therapy is effective. Although oxygen saturation and ABGs are monitored in patient's receiving hyperventilation, they do not provide data about whether the therapy is successful in reducing ICP. Breath sounds are assessed, but they are not helpful in determining whether the hyperventilation is effective. Cognitive Level: Application Text Reference: p. 1475 Nursing Process: Evaluation NCLEX: Physiological Integrity

The term "pink puffer" generally refers to which pulmonary disease? A. Adult respiratory distress syndrome (ARDS) B. Asthma C. Chronic obstructive disease D. Emphysema

D. Emphysema Because of the large amount of energy it takes to breathe, clients with emphysema are usually cachectic. They're pink & usually breathe through pursed lips, hence the term "puffer".

Which option indicates a sign of Cushing's triad, an indication of increased intracranial pressure (ICP)? A. Heart rate increases from 90 to 110 beats/minute B. Kussmaul respirations C. Temperature over 100.4° F (38° C) D. Heart rate decreases from 75 to 55 beats/minute

D Cushing's triad is systolic hypertension with a widening pulse pressure, bradycardia with a full and bounding pulse, and slowed respirations. The rise in blood pressure is an attempt to maintain cerebral perfusion, and it is a neurologic emergency because decompensation is imminent. The other options are not part of Cushing's triad.

Which option is the most sensitive indication of increased ICP? A. Papilledema B. Cushing's triad C. Projectile vomiting D. Change in the level of consciousness (LOC)

D The LOC is the most sensitive and reliable indicator of the patient's neurologic status. Changes in LOC are a result of impaired cerebral brain flow. Papilledema and Cushing's triad are late signs. Projectile vomiting is not a sensitive indicator.

An emergency department nurse is caring for a client who has died from a suspected homicide. Which action should the nurse take? a. Remove all tubes and wires in preparation for the medical examiner. b. Limit the number of visitors to minimize the family's trauma. c. Consult the bereavement committee to follow up with the grieving family. d. Communicate the client's death to the family in a simple and concrete manner.

D ~ When dealing with client's and families in crisis, communicate in a simple and concrete manner to minimize confusion. Tubes must remain in place for the medical examiner. Family should be allowed to view the body. Offering to call for additional family support during the crisis is suggested. The bereavement committee should be consulted, but this is not the priority at this time.

Which of the following pathophysiological mechanisms that occur in the lung parenchyma allows pneumonia to develop? A. Atelectasis B. Bronchiectasis C. Effusion D. Inflammation

D. Inflammation

A nurse prepares to discharge an older adult client home from the emergency department (ED). Which actions should the nurse take to prevent future ED visits? (SATA) a. Provide medical supplies to the family. b. Consult a home health agency. c. Encourage participation in community activities. d. Screen for depression and suicide. e. Complete a functional assessment.

D, E ~ Due to the high rate of suicide among older adults, a nurse should assess all older adults for depression and suicide. The nurse should also screen older adults for functional assessment, cognitive assessment, and risk for falls to prevent future ED visits.

A client is in danger of respiratory arrest following the administration of a narcotic analgesic. An arterial blood gas value is obtained. The nurse would expect to PaCO2 to be which of the following values? A. 15 mm Hg B. 30 mm Hg C. 40 mm Hg D. 80 mm Hg

D. 80 mm Hg A client about to go into respiratory arrest will have inefficient ventilation & will be retaining CO2.

A patient with ICP monitoring has pressure of 12 mm Hg. The nurse understand that this pressure reflects a. a severe decrease in cerebral perfusion pressure b. an alteration in the production of CSF c. the loss of autoregulatory control of ICP d. a normal balance between brain tissue, blood, and CSF

D. A normal balance between brain tissue, blood, and CSF- normal is 10- 15 mm Hg

For which patient would NIPPV be an appropriate intervention to promote oxygenation? A. A patient's whose cardiac output and blood pressure are unstable B. A patient whose respiratory failure is caused by a head injury with loss of consciousness C. A patient with a diagnosis of cystic fibrosis and who is producing copious secretions D. A patient who is experiencing respiratory failure as a result of the progression of myasthenia gravis

D. A patient who is experiencing respiratory failure as a result of the progression of myasthenia gravis NIPPV is most effective in treating patients with respiratory failure due to chest wall and neuromuscular disease. It is not recommended for patients who are experiencing cardiac instability, decreased level of consciousness, or excessive secretions.

When assessing the body function of a patient with increased ICP, the nurse should initially assess a. corneal reflex testing b. extremity strength testing c. pupillary reaction to light d. circulatory and respiratory status

D. Circulatory and respiratory status- Of the body functions that should be assessed in an unconscious patient, cardiopulmonary status is the most vital function and gives priorities to the ABCs (airway, breathing, and circulation)

A 66 year old client has marked dyspnea at rest, is thin, uses accessory muscles to breathe. He's tachypneic, with a prolonged expiratory phase. He has no cough. He leans forward with his arms braced on his knees to support his chest & shoulders for breathing. This client has symptoms of which of the following disorders? A. Adult respiratory distress syndrome (ARDS) B. Asthma C. Chronic obstructive disease D. Emphysema

D. Emphysema

A 69 year old client appears thin & cachectic. He's short of breath at rest & his dyspnea increases with the slightest exertion. His breath sounds are diminished, even with deep inspiration. These signs & symptoms fit which of the following conditions? A. Adult respiratory distress syndrome (ARDS) B. Asthma C. Chronic obstructive disease D. Emphysema

D. Emphysema

Nurses in change-of-shift report are discussing the care of a patient with a stroke who has progressively increasing weakness and decreasing level of consciousness (LOC). Which nursing diagnosis do they determine has the highest priority for the patient? a. Impaired physical mobility related to weakness b. Disturbed sensory perception related to brain injury c. Risk for impaired skin integrity related to immobility d. Risk for aspiration related to inability to protect airway

D. Protection of the airway is the priority of nursing care for a patient having an acute stroke. The other diagnoses are also appropriate, but interventions to prevent aspiration are the priority at this time.

It's highly recommended that clients with asthma, chronic bronchitis, and emphysema have Pneumovax and flu vaccinations for which of the following reasons? A. All clients are recommended to have these vaccines B. These vaccines produce bronchodilation and improve oxygenation C. These vaccines help reduce the tachypnea these clients experience D. Respiratory infections can cause severe hypoxia and possibly death in these clients

D. Respiratory infections can cause severe hypoxia & possible death in these clients

A 73-year-old patient with a stroke experiences facial drooping on the right side and right-sided arm and leg paralysis. When admitting the patient, which clinical manifestation will the nurse expect to find? a. Impulsive behavior b. Right-sided neglect c. Hyperactive left-sided tendon reflexes d. Difficulty comprehending instructions

D. Right-sided paralysis indicates a left-brain stroke, which will lead to difficulty with comprehension and use of language. The left-side reflexes are likely to be intact. Impulsive behavior and neglect are more likely with a right-side stroke.

Which patient is having the most difficulty breathing? A. The patient who reports one-pillow orthopnea B. The patient with an inspiratory to expiratory ratio of 1:2 C. The patient who speaks a sentence before breathing D. The patient with paradoxic breathing

D. The patient with paradoxic breathing Paradoxic breathing indicates severe distress. The thorax and abdomen normally move outward on inspiration and inward on exhalation. During paradoxic breathing, the abdomen and chest move in the opposite manner, and the pattern results from maximal use of the accessory muscles of respiration. Orthopnea, measured by the number of pillows needed to breathe comfortably, is associated with the use of one to four pillows. One pillow indicates a minor condition. Normal inspiratory to expiratory ratio is 1:2. Speaking in sentences before having to take a breath indicates mild or no distress.

A 56-year-old patient arrives in the emergency department with hemiparesis and dysarthria that started 2 hours previously, and health records show a history of several transient ischemic attacks (TIAs). The nurse anticipates preparing the patient for a. surgical endarterectomy. b. transluminal angioplasty. c. intravenous heparin administration. d. tissue plasminogen activator (tPA) infusion.

D. The patient's history and clinical manifestations suggest an acute ischemic stroke and a patient who is seen within 4.5 hours of stroke onset is likely to receive tPA (after screening with a CT scan). Heparin administration in the emergency phase is not indicated. Emergent carotid transluminal angioplasty or endarterectomy is not indicated for the patient who is having an acute ischemic stroke.

Skull radiographs and a computed tomography (CT) scan provide evidence of a depressed parietal fracture with a subdural hematoma in a patient admitted to the emergency department following an automobile accident. In planning care for the patient, the nurse anticipates that a. the patient will receive life-support measures until the condition stabilizes b. immediate burr holes will be made to rapidly decompress the intracranial activity c. the patient will be treated conservatively with close monitoring for changes in neurologic condition d. the patient will be taken to surgery for a craniotomy for evacuation of blood and decompression of the cranium

D. When there is a depressed fracture and fractures with loose fragments, a craniotomy is indicated to elevate the depressed bone and remove free fragments. A craniotomy is also indicated in cases of acute subdural and epidural hematomas to remove the blood and control the bleeding. Burr holes may be used in an extreme emergency for rapid decompression, but with a depressed fracture, surgery would be the treatment of choice

What is the classic chest x-ray finding in a patient with late-stage ARDS? A. Hyperinflation B. Infiltrates in the bases C. Deflated lung on one side D. White lung

D. White lung The chest x-ray image often shows a white-out effect, sometimes called white lung, because consolidation and infiltrates are widespread throughout the lungs, leaving few recognizable air spaces. Hyperinflation typically is seen in chronic obstructive lung disease. There can be some effusions or infiltrates initially with ARDS, but it is more extensive in the late stages and is not only in the bases. Unilateral lung deflation is pneumothorax.

You're providing care to a patient who was just transferred to your unit for the treatment of ARDS. The patient is in the exudative phase. The patient is ordered arterial blood gases. The results are back. Which results are expected during this early phase of acute respiratory distress syndrome that correlates with this diagnosis? A. PaO2 40, pH 7.59, PaCO2 30, HCO3 23 B. PaO2 85, pH 7.42, PaCO2 37, HCO3 26 C. PaO2 50, pH 7.20, PaCO2 48, HCO3 29 D. PaO2 55, pH 7.26, PaCO2 58, HCO3 19

The answer is A. This option demonstrates respiratory alkalosis. In the early stages of ARDS (exudative) the patient will start to enter in respiratory alkalosis. The patient starts to have tachypnea (the body's way of trying to increase the oxygen level but it can't). They will have a very low PaO2 level (normal PaO2 is 80 mmHg), the blood pH will become high (normal is 7.35-7.45) (alkalotic). In the late stage, the patient can enter into respiratory acidosis.

During the exudative phase of acute respiratory distress syndrome (ARDS), the patient's lung cells that produce surfactant have become damaged. As the nurse you know this will lead to? A. bronchoconstriction B. atelectasis C. upper airway blockage D. pulmonary edema

The answer is B. Surfactant decreases surface tension in the lungs. Therefore, the alveoli sacs will stay stable when a person exhales (hence the sac won't collapse). If there is a decrease in surfactant production this creates an unpredictable alveoli sac that can easily collapse, hence a condition called ATELETASIS will occur (collapse of the lung tissue) when there is a decrease production in surfactant.

You're providing care to a patient who is being treated for aspiration pneumonia. The patient is on a 100% non-rebreather mask. Which finding below is a HALLMARK sign and symptom that the patient is developing acute respiratory distress syndrome (ARDS)? A. The patient is experiencing bradypnea. B. The patient is tired and confused. C. The patient's PaO2 remains at 45 mmHg. D. The patient's blood pressure is 180/96.

The answer is C. A hallmark sign and symptom found in ARDS is refractory hypoxemia. This is where that although the patient is receiving a high amount of oxygen (here a 100% non-rebreather mask) the patient is STILL hypoxic. Option C is the answer because it states the patient's arterial oxygen level is remaining at 45 mmHg (a normal is 80 mmHg but when treating patients with ARDS a goal is at least 60 mmHg).

Which patient below is at MOST risk for developing ARDS and has the worst prognosis? A. A 52-year-old male patient with a pneumothorax. B. A 48-year-old male being treated for diabetic ketoacidosis. C. A 69-year-old female with sepsis caused by a gram-negative bacterial infection. D. A 30-year-old female with cystic fibrosis

The answer is C. Sepsis is the MOST common cause of ARDS because of systemic inflammation experienced. This is also true if the cause of the sepsis is a gram-negative bacterium (this also makes the infection harder to treat...hence poor prognosis). With sepsis, the immune cells that are present with the inflammation travel to the lungs and damage the alveolar capillary membrane leading to fluid to leak in the alveolar sacs.

A patient has been hospitalized in the ICU for a near drowning event. The patient's respiratory function has been deteriorating over the last 24 hours. The physician suspects acute respiratory distress syndrome. A STAT chest x-ray is ordered. What finding on the chest x-ray is indicative of ARDS? A. infiltrates only on the upper lobes B. enlargement of the heart with bilateral lower lobe infiltrates C. white-out infiltrates bilaterally D. normal chest x-ray

The answer is C. This is a finding found in ARDS....pronounce white-out infiltrates bilaterally.

A patient is experiencing respiratory failure due to pulmonary edema. The physician suspects ARDS but wants to rule out a cardiac cause. A pulmonary artery wedge pressure is obtained. As the nurse you know that what measurement reading obtained indicates that this type of respiratory failure is NOT cardiac related? A. >25 mmHg B. <10 mmHg C. >50 mmHg D. <18 mmHg

The answer is D. A pulmonary artery wedge pressure measures the left atrial pressure. A pulmonary catheter is "wedged" with a balloon in the pulmonary arterial branch to measure the pressure. If the reading is less than 18 mmHg it indicates this is NOT a cardiac issue but most likely ARDS. Therefore, the pulmonary edema is due to damage to the alveolar capillary membrane leaking fluid into the alveolar sac....NOT a heart problem ex: heart failure.

You are caring for a patient with acute respiratory distress syndrome. As the nurse you know that prone positioning can be beneficial for some patients with this condition. Which findings below indicate this type of positioning was beneficial for your patient with ARDS? SATA A. Improvement in lung sounds B. Development of a V/Q mismatch C. PaO2 increased from 59 mmHg to 82 mmHg D. PEEP needs to be titrated to 15 mmHg of water

The answers are A and C. Prone positioning helps improve PaO2 (82 mmHg is a good finding) without actually giving the patient high concentrations of oxygen. It helps improves perfusion and ventilation (hence correcting the V/Q mismatch). In this position, the heart is no longer laying against the posterior part of the lungs (improves air flow...hence improvement of lung sounds) and it helps move secretions from other areas that were fluid filled and couldn't move in the supine position, hence helping improve atelectasis.

A nurse reviews laboratory results for a client who was admitted for a myocardial infarction and cardiogenic shock 2 days ago. Which laboratory test result should the nurse expect to find? a. Blood urea nitrogen (BUN) of 52 mg/dL b. Creatinine of 2.3 mg/dL c. BUN of 10 mg/dL d. BUN/creatinine ratio of 8:1

a. Blood urea nitrogen (BUN) of 52 mg/dL Shock leads to decreased renal perfusion. An elevated BUN accompanies this condition. The creatinine should be normal because no kidney damage occurred. A low BUN signifies overhydration, malnutrition, or liver damage. A low BUN/creatinine ratio indicates fluid volume excess or acute renal tubular acidosis.

A client is hospitalized after a myocardial infarction. Which hemodynamic parameters does the nurse correlate with cardiogenic shock? (Select all that apply.) a. Decreased cardiac output b. Increased cardiac output c. Increased mean arterial pressure (MAP) d. Decreased MAP e. Increased afterload f. Decreased afterload

a. Decreased cardiac output d. Decreased MAP e. Increased afterload Myocardial infarction (MI) is a major cause of direct pump failure. With MI, cardiac output and MAP are decreased and afterload is increased. The other parameters do not correlate with pump failure

The nurse is caring for a patient admitted for evaluation and surgical removal of a brain tumor. Which complications will the nurse monitor for (select all that apply.)? Select all that apply. a. Seizures b. Vision loss c. Cerebral edema d. Pituitary dysfunction e. Parathyroid dysfunction f. Focal neurologic deficits

a. Seizures b. Vision loss c. Cerebral edema d. Pituitary dysfunction f. Focal neurologic deficits Brain tumors can cause a wide variety of symptoms depending on location such as seizures, vision loss, and focal neurologic deficits. Tumors can put pressure on the pituitary, leading to dysfunction of the gland. As the tumor grows, clinical manifestations of increased intracranial pressure and cerebral edema appear. The parathyroid gland is not regulated by the cerebral cortex or the pituitary gland.

The patient's magnetic resonance imaging revealed the presence of a brain tumor. The nurse anticipates which treatment modality? a. Surgery b. Chemotherapy c. Radiation therapy d. Biologic drug therapy

a. Surgery Surgical removal is the preferred treatment for brain tumors. Chemotherapy and biologic drug therapy are limited by the blood-brain barrier, tumor cell heterogeneity, and tumor cell drug resistance. Radiation therapy may be used as a follow-up measure after surgery.

A patient with a myocardial infarction (MI) and cardiogenic shock has the following vital signs: BP 86/50, pulse 126, respirations 30. Hemodynamic monitoring reveals an elevated PAWP and decreased cardiac output. The nurse will anticipate: a.) administration of furosemide (Lasix) IV. b.) titration of an epinephrine (Adrenalin) drip. c.) administration of a normal saline bolus. d.) assisting with endotracheal intubation.

a.) administration of furosemide (Lasix) IV. Rationale: The PAWP indicates that the patient's preload is elevated and furosemide is indicated to reduce the preload and improve cardiac output. Epinephrine would further increase myocardial oxygen demand and might extend the MI. The PAWP is already elevated, so normal saline boluses would be contraindicated. There is no indication that the patient requires endotracheal intubation.

A patient with increased ICP is being monitored in the intensive care unit (ICU) with a fiberoptic catheter. Which order is a priority for you? A. Perform hourly neurologic checks. B. Take a complete set of vital signs. C. Administer the prescribed mannitol (Osmitrol). D. Give an H2-receptor blocker.

c he priority is to treat the known existing problem, and mannitol is the only thing that can do that. Because the patient is having the current pressure measured with objective numbers, treating the known problem is a priority over additional assessments. H2-blockers are given when corticosteroids are administered to help prevent gastrointestinal bleeding, but they are not a priority compared with the treatment of ICP.

The nurse is caring for a patient in cardiogenic shock who is being treated with an infusion of dobutamine (Dobutrex). The physician's order calls for the nurse to titrate the infusion to achieve a cardiac index of >2.5 L/min/m2 . The nurse measures a cardiac output, and the calculated cardiac index for the patient is 4.6 L/min/m2 . What is the best action by the nurse? a. Obtain a stat serum potassium level. b. Order a stat 12-lead electrocardiogram. c. Reduce the rate of dobutamine (Dobutrex). d. Assess the patient's hourly urine output.

c. Reduce the rate of dobutamine (Dobutrex). Dobutamine (Dobutrex) is used to stimulate contractility and heart rate while causing vasodilation in low cardiac output states improving overall cardiac performance. The patient's cardiac index is well above normal limits, so the rate of infusion of the medication should be reduced so as not to overstimulate the heart. There is no evidence to support the need for a serum potassium or 12-lead electrocardiogram. Assessment of hourly urine output is important in the care of the patient in cardiogenic shock, but it is not a priority in this scenario.


Related study sets

Unit 8: Post WWII & The Cold War

View Set

Biology Section 3-2 Review: Molecules of Life

View Set

Scientific Inquiry Test Review (5th Grade)

View Set

World War 1 And Spanish-American War

View Set